Ads Code

No title

2024-08-02T06:25:06+05:30
Question:
'आर्य समाज ’के संस्थापक कौन थे?
  • 71% answered correctly
  • Your first attempt
  • Your first attempt
Re-attempt answer
Click here to see the answer now
Reattempt mode is Off. Turn it on from bottom bar

Your First Attempt Answers

  • Accepted answer is between: 0 and 0
  • सही उत्तर है स्वामी दयानंद सरस्वती

    Key Points

    • आर्य समाज भारत में एक एकेश्वरवादी हिंदू सुधार आंदोलन है जो वेदों के अचूक अधिकार में विश्वास के आधार पर सिद्धांतों और प्रथाओं को बढ़ावा देता है।
    • महर्षि दयानंद सरस्वती ने 10 अप्रैल, 1875 को आर्य समाज की स्थापना की।
    • आर्य समाज इस मान्यता का पालन करता है कि केवल एक ही ईश्वर है और उसकी मूर्ति पूजा उचित नहीं है।
    • आर्य समाज अभियोजन को बढ़ावा देने वाला पहला हिंदू संगठन था।

    Important Points

    • दयानंद सरस्वती एक हिंदू विद्वान, सामाजिक कार्यकर्ता और आर्य समाज के संस्थापक, वैदिक धर्म सुधार आंदोलन के संस्थापक थे।
    • 1876 ​​में, उन्होंने स्वराज के लिए पहली बार भारत को "भारतीयों के लिए भारत" के रूप में संदर्भित किया, जिसे लोकमान्य तिलक ने बाद में अपनाया।

    Additional Information

    • राजा राम मोहन राय
      • वह ब्रह्म सभा के संस्थापकों में से एक थे, जो कि भारतीय उपमहाद्वीप में सामाजिक-धार्मिक सुधार आंदोलन, ब्रह्म समाज के अग्रदूत थे।
    • स्वामी विवेकानंद
      • वह एक भारतीय हिंदू संत थे, जो 18 वीं शताब्दी से 20 वीं शताब्दी के प्रारंभ तक रहे। वह उन्नीसवीं सदी में भारतीय रहस्यवादी रामकृष्ण के करीबी अनुयायी थे।
    • रामकृष्ण मठ और रामकृष्ण मिशन दोनों की स्थापना स्वामी विवेकानंद ने की थी।
    Was the solution helpful?
    Question:

    उस विकल्प का चयन करें जो दिए गए चित्र में त्रिभुजों की संख्या को दर्शाता है।

    • 40% answered correctly
    • Your first attempt
    • Your first attempt
    Re-attempt answer
    Click here to see the answer now
    Reattempt mode is Off. Turn it on from bottom bar

    Your First Attempt Answers

  • Accepted answer is between: 0 and 0
  • बनने वाले त्रिभुजों की संख्या इस प्रकार है:

    इस प्रकार, त्रिभुजों की कुल संख्या 18 है।

    अत:सही उत्तर "18" है।

    Question:
    हमें बोमेन संपुट कहाँ मिल सकता है?
    • 25% answered correctly
    • Your first attempt
    • Your first attempt
    Re-attempt answer
    Click here to see the answer now
    Reattempt mode is Off. Turn it on from bottom bar

    Your First Attempt Answers

  • Accepted answer is between: 0 and 0
  • सही उत्तर नेफ्रॉन के अंत में है।

    अवधारणा:

    उत्सर्जन तंत्र:

    • उत्सर्जन शरीर से अपशिष्ट और अतिरिक्त जल त्यागने की प्रक्रिया है।
    • उत्सर्जन के अंगों में त्वचा, यकृत, बड़ी आंत, फेफड़े और वृक्क शामिल हैं। ये अंग मिलकर उत्सर्जन तंत्र बनाते हैं।

    स्पष्टीकरण:

    वृक्क:

    • युग्मित वृक्क को अक्सर उत्सर्जन का मुख्य अंग माना जाता है।
    • वृक्क का प्राथमिक कार्य मूत्र के रूप में तरल अपशिष्ट के उत्पादन द्वारा रक्तप्रवाह से अतिरिक्त जल और अपशिष्ट को खत्म करना है।
    • वृक्क की मुख्य संरचनात्मक और कार्यात्मक इकाइयाँ छोटी संरचनाएँ होती हैं जिन्हें नेफ्रॉन (वृक्काणु) कहा जाता है।
    • नेफ्रॉन रक्त से पदार्थों का निस्पंद करता हैं, जो आवश्यक है उसे रक्त में वापस कर देते हैं और बाकी को मूत्र के रूप में बाहर निकाल देते हैं।
    • वृक्क, मूत्र तंत्र के अंग हैं, जिसमें मूत्रवाहिनी, मूत्राशय और मूत्रमार्ग भी शामिल हैं। ये अंग क्रमशः मूत्र का परिवहन, संग्रह करते हैं और उसे बाहर निकाल देते हैं।

    नेफ्रॉन की संरचना:

    • यह संबंधित रक्त वाहिकाओं की श्रृंखला वाली एक लंबी नलिका है।
    • नलिका का ऊपरी सिरा एक वर्धित कप के आकार की संरचना है जिसे बोमेन संपुट कहा जाता है।
    • बोमेन संपुट के नीचे, नलिका कॉइल समीपस्थ नलिका बनाती है और फिर यह एक हेयरपिन टर्न का अनुसरण करती है जिसे हेनले लूप कहा जाता है।
    • हेनले लूप के बाद नलिका एक बार फिर दूरस्थ नलिका के रूप में कुंडलित हो जाती है।
    • यह फिर एक संग्रह नलिका में प्रवेश करती है, जो अन्य दूरस्थ नलिकाओं से भी मूत्र प्राप्त करती है।
    • बोमेन संपुट में केशिकाओं की एक कुंडलित गेंद होती है जिसे केशिकागुच्छ के रूप में जाना जाता है।

    इस प्रकार, बोमेन संपुट नेफ्रॉन के अंत में पाया जाता है।

    Question:
    अनुच्छेद 300 ए के तहत निम्नलिखित में से कौन सा अधिकार आता है?
    • 77% answered correctly
    • Your first attempt
    • Your first attempt
    Re-attempt answer
    Click here to see the answer now
    Reattempt mode is Off. Turn it on from bottom bar

    Your First Attempt Answers

  • Accepted answer is between: 0 and 0
  • सही उत्तर संपत्ति का अधिकार है।


    प्रमुख बिंदु

    • आर्टिकल 300a
      • अनुच्छेद 300ए संपत्ति के अधिकार से संबंधित है।
      • अनुच्छेद 300a कहता है कि किसी भी व्यक्ति को उसकी संपत्ति से वंचित नहीं किया जाएगा, सिवाय कानून के।
      • संपत्ति के अधिकार को मूल अधिकार से 1978 के 44वें संविधान संशोधन अधिनियम के बाद अस्वीकार कर दिया गया था।
      • संशोधन के बाद, इसे संविधान के भाग-XII में अनुच्छेद 300a के तहत कानूनी अधिकार दिया गया था।
      • सुप्रीम कोर्ट ने सरकार को याद दिलाया कि किसी व्यक्ति की निजी संपत्ति लेने से पहले राज्य को कानून की उचित प्रक्रिया का पालन करना होगा।

    अतिरिक्त जानकारी

    स्वतंत्रता का अधिकार यह अनुच्छेद 19 से 22 के तहत व्यक्तिगत अधिकारों की गारंटी देता है
    जीवन का अधिकार यह सुनिश्चित करता है कि सरकार सहित कोई भी आपके जीवन को समाप्त करने की कोशिश नहीं कर सकता, अनुच्छेद 21 के तहत
    शिक्षा का अधिकार

     अधिनियम का उद्देश्य 6 वर्ष से 14 वर्ष की आयु के बच्चों की अनिवार्य शिक्षा का लक्ष्य है। अनुच्छेद 21-ए के तहत

    Question:

    Choose the correct forms of the modal verb:

    I _____ win the race.

    • 56% answered correctly
    • Your first attempt
    • Your first attempt
    Re-attempt answer
    Click here to see the answer now
    Reattempt mode is Off. Turn it on from bottom bar

    Your First Attempt Answers

  • Accepted answer is between: 0 and 0
  • The correct answer is 'must'.

     Key Points

    • The sentence is expressing a necessity or obligation, which requires the use of the modal verb 'must'.
    • Options 1 ('do') and 4 ('has') are not modal verbs and do not fit in the sentence.
    • Option 3 ('does not') is also inappropriate as it implies a negative, while the sentence is making a positive statement about the speaker's determination to win.
    • The modal verb 'must' conveys the speaker's obligation or strong determination to win the race.

    Therefore, the correct answer is Option 2.

    Question:

    Choose the word that can substitute the given sentence.

    One who can use either hand with ease.

    • 48% answered correctly
    • Your first attempt
    • Your first attempt
    Re-attempt answer
    Click here to see the answer now
    Reattempt mode is Off. Turn it on from bottom bar

    Your First Attempt Answers

  • Accepted answer is between: 0 and 0
  • The correct answer is “Ambidextrous”.

     Key Points

    • 'One who can use either hand with ease' refers to a person who has the ability to use both the right and left hand equally well. -This can mean that in Hindi - दोनों हाथों का समान रूप से उपयोग करने में सक्षम व्यक्ति.
    • 'Ambidextrous' is a term used to describe someone who can use either hand with equal ease. -This can mean that in Hindi - कोई व्यक्ति जो दोनों हाथों का समान रूप से उपयोग कर सकता है.
    • Example Sentence for the given Phrase: His ability to write with either hand demonstrated that he is one who can use either hand with ease.
    • Example Sentence for "Ambidextrous": The artist was ambidextrous, able to paint skillfully with both his left and right hand.
    • Hence, the option that is similar in meaning to the given phrase is 'Ambidextrous'.

    Therefore, the correct answer is ‘Option 2’.

    Additional Information

    • Let’s have a look at the meaning of the remaining options. 
    • 'Connoisseur' is a term for someone who has expert knowledge in a particular field. (किसी विशेष क्षेत्र में विशेषज्ञ ज्ञान वाला।)
    • 'Fugitive' is used to describe a person who has escaped from captivity or is in hiding. (ऐसा व्यक्ति जो कैद से भाग गया हो या छिपा हुआ हो।)
    • 'Ascetic' describes a person who practices severe self-discipline and refrains from indulging in any forms of worldly pleasure. (ऐसा व्यक्ति जो कठोर आत्म-अनुशासन का पालन करता है और सांसारिक सुखों में भोग करने से बचता है।)
    Question:

    In the following question, out of the given four alternatives, select the alternative which best expresses the meaning of the Idiom/Phrase.

    Fit as a fiddle 

    • 51% answered correctly
    • Your first attempt
    • Your first attempt
    Re-attempt answer
    Click here to see the answer now
    Reattempt mode is Off. Turn it on from bottom bar

    Your First Attempt Answers

  • Accepted answer is between: 0 and 0
  • The correct answer is 'In a very good physical condition '.

    Key Points

    • The idiom "Fit as a fiddle" best expresses the meaning of: 'In a very good physical condition'
    • "Fit as a fiddle" is an idiom used to describe someone who is in excellent health or physical condition.
    • "In a very good physical condition": This refers to someone who is healthy and physically fit. (बहुत अच्छी शारीरिक स्थिति में)

    Therfore, the correct answer is: 'In a very good physical condition'.

    Additional Information

    • Now, let's explain all the given options along with their English and Hindi meanings:
      • "To be in love": This refers to the emotional state of deep affection for someone.(प्यार में होना)
      • "To drink alcohol regularly": This means the consistent consumption of alcoholic beverages.(नियमित रूप से शराब पीना)
      • "To be firm": This refers to someone who is steadfast, resolute, or unwavering in their beliefs or decisions.(दृढ़ होना)
    Question:
    मौलिक कर्तव्य भारतीय संविधान के किस भाग में है?
    • 80% answered correctly
    • Your first attempt
    • Your first attempt
    Re-attempt answer
    Click here to see the answer now
    Reattempt mode is Off. Turn it on from bottom bar

    Your First Attempt Answers

  • Accepted answer is between: 0 and 0
  • भारतीय संविधान ने इसे देश के प्रत्येक नागरिक को मौलिक अधिकार दिए हैं।

    • इसलिए, सभी को अपने संविधान और देश की रक्षा करनी चाहिए।
    • मौलिक कर्तव्य वे कर्तव्य हैं जिन्हें प्रत्येक व्यक्ति को निभाने की आवश्यकता होती है। हालाँकि, यह एक व्यक्ति का अधिकार है यदि वह मौलिक कर्तव्यों का पालन करना चाहता है।
    • मौलिक कर्तव्यों की मुख्य विशेषताएं इस प्रकार हैं:
      • सरदार स्वर्ण सिंह समिति ने इसकी अनुशंसा की थी।
      • इसे भारतीय संविधान में 42वें संशोधन के बाद जोड़ा गया, जिसे लघु संविधान भी कहा जाता है।
      • यह भाग 4A में अनुच्छेद 51-A के तहत बताई गई संख्या में 11 है।

    अत:, हम यह निष्कर्ष निकालते हैं कि मौलिक कर्तव्य भारतीय संविधान के भाग IV-A में हैं। 

    Question:
    भारतीय प्रतिस्पर्धा आयोग के अध्यक्ष के रूप में किसे नियुक्त किया गया है?
    • 31% answered correctly
    • Your first attempt
    • Your first attempt
    Re-attempt answer
    Click here to see the answer now
    Reattempt mode is Off. Turn it on from bottom bar

    Your First Attempt Answers

  • Accepted answer is between: 0 and 0
  • सही उत्तर रवनीत कौर है।

    In News

    • रवनीत कौर को मई 2023 में भारतीय प्रतिस्पर्धा आयोग का अध्यक्ष नियुक्त किया गया।

    Key Points

    • केंद्र सरकार ने 1988 की पंजाब कैडर की IAS अधिकारी रवनीत कौर को भारतीय प्रतिस्पर्धा आयोग का नया अध्यक्ष नियुक्त किया।

    Centre appoints Ravneet Kaur as CCI Chairperson - The Hindu BusinessLine

    • वह वर्तमान में चंडीगढ़ में राजस्व, पुनर्वास और आपदा प्रबंधन विभाग में अतिरिक्त मुख्य सचिव और वित्तीय आयुक्त हैं।
    • संगीता वर्मा कार्यकारी अध्यक्ष के रूप में कार्यरत हैं।
    • 15 मई के आदेश के अनुसार, उनकी नियुक्ति पदभार ग्रहण करने की तिथि से पांच वर्ष या 65 वर्ष की आयु प्राप्त करने की तिथि तक या अगले आदेश तक, जो भी पहले हो, तक होगी।

    Additional Information

    • प्रवीण सूद को CBI निदेशक नियुक्त किया गया है।
    • मनोज सोनी को UPSC का अध्यक्ष नियुक्त किया गया है।
    • आईपीएस अधिकारी रश्मी शुक्ला को पहले सशस्त्र सीमा बल के महानिदेशक के रूप में नियुक्त किया गया था।
    • उनका वर्तमान पदनाम महाराष्ट्र के महानिदेशक का है।
    Question:
    विद्युत धारा में प्रतिरोध के श्रेणीक्रम संयोजन में, निम्नलिखित में से क्या सही है?
    • 54% answered correctly
    • Your first attempt
    • Your first attempt
    Re-attempt answer
    Click here to see the answer now
    Reattempt mode is Off. Turn it on from bottom bar

    Your First Attempt Answers

  • Accepted answer is between: 0 and 0
  • सही उत्तर है प्रत्येक प्रतिरोध के माध्यम से प्रवाहित विद्युत धारा समान होती है।Key Points

    • जब एक ही समय में प्रत्येक प्रतिरोधक से समान मात्रा में विद्युत धारा प्रवाहित होती है, तब ये कहा जाता है कि दो या दो से अधिक प्रतिरोधकों को श्रेणी में जोड़ा गया है।
    • ऐसे परिपथ में प्रत्येक प्रतिरोधक में वोल्टेज भिन्न होता है।
    • यदि किसी श्रेणी में कोई प्रतिरोधक कनेक्शन टूट जाता है या कोई समस्या होती है, तब परिपथ पूरी तरह से बंद हो जाता है।
    • Rtotal = R1 + R2 + ..... + Rn
    • सभी व्यक्तिगत प्रतिरोधों का योग निकाय के समग्र प्रतिरोध को बनाता है।

    Additional Information

    • जब वोल्टेज सभी प्रतिरोधकों में समान होता है, तब दो या दो से अधिक प्रतिरोधकों को समानांतर में जोड़ा गया होता है।
    • जब ऐसे परिपथों में शाखाएँ एक सामान्य स्थान पर एक साथ आती हैं, तो विद्युत धारा विभाजित हो जाती है और फिर पुनः संयोजित हो जाती है।
    • एक समानांतर परिपथ में, अन्य घटकों को प्रभावित किए बिना एक प्रतिरोधक या किसी अन्य घटक को जोड़ना या अलग करना आसान है।
    • निकाय का समग्र व्युत्क्रम प्रतिरोध प्रत्येक प्रतिरोधक के पारस्परिक योग के बराबर होता है।
    Question:
    'फैरड' किसकी इकाई है?
    • 65% answered correctly
    • Your first attempt
    • Your first attempt
    Re-attempt answer
    Click here to see the answer now
    Reattempt mode is Off. Turn it on from bottom bar

    Your First Attempt Answers

  • Accepted answer is between: 0 and 0
  • संकल्पना:

    धारिता

    • धारिता विद्युत चालक का एक गुण है जिसे अलग-अलग विद्युत आवेश की मात्रा से मापा जाता है जिसे विद्युत विभव में प्रति यूनिट परिवर्तन पर संग्रहीत किया जा सकता है।
    • संधारित्र में, आवेश की मात्रा, Q = CV, जहाँ, C = धारिता, V = विद्युत विभव
    • एक विद्युत परिपथ में एक धारिता को एक संधारित्र नामक उपकरण द्वारा पुन:स्थापित किया जाता है।
    • धारिता की SI इकाई फैरड है जिसे F द्वारा निरूपित किया जाता है।

    व्याख्या:

    धारिता की SI इकाई फैरड है जिसे F द्वारा निरूपित किया जाता है।

    Additional Information

    • प्रतिरोध
      • ​​विद्युत परिपथ में प्रतिरोध को एक प्रतिरोध नामक उपकरण द्वारा पुन:स्थापित किया जाता है।
      • सूत्र, V = IR, जहां I = परिपथ में प्रवाहित विद्युत धारा, V = आपूर्ति वोल्टेज, R = प्रतिरोध
      • प्रतिरोध की SI इकाई ओम है जिसे Ω द्वारा निरूपित किया जाता है।
    • चालकत्‍व
      • किसी चालक में प्रतिरोध का व्युत्क्रम चालकत्‍व कहलाता है
      • संबंध, Conductance=1Resistance
      • चालकत्‍व की SI इकाई ओम-1 है।
    • प्रेरकत्व
      • एक विद्युत परिपथ में प्रेरकत्व एक प्रेरक नामक उपकरण द्वारा पुन:स्थापित किया जाता है।
      • प्रेरकत्व की SI इकाई हेनरी है जिसे H द्वारा निरूपित किया जाता है।
    Question:
    निम्नलिखित में से कौन सा देश मकर रेखा से गुजरता है?
    • 75% answered correctly
    • Your first attempt
    • Your first attempt
    Re-attempt answer
    Click here to see the answer now
    Reattempt mode is Off. Turn it on from bottom bar

    Your First Attempt Answers

  • Accepted answer is between: 0 and 0
  • सही उत्तर उपरोक्त सभी है

    Key Points

    • मकर रेखा (Tropic of Capricorn) 23 ° 26.612.6 the पर स्थित दक्षिणी अक्षांश है यह दक्षिणी गोलार्ध के कर्क रेखा के समतुल्य है
    • मकर रेखा ट्रोपिक ऑस्ट्रेलिया, चिली, दक्षिणी ब्राजील और उत्तरी दक्षिण अफ्रीका से होकर गुजरती है
    • ब्राज़ील एकमात्र ऐसा देश है जो मकर रेखा के भूमध्य रेखा और ट्रोपिक दोनों से होकर गुजरता है

     

    Question:

    एक व्यक्ति 'A' किसी बिंदु से चलना प्रारम्भ करता है और 2 किमी उत्तर की ओर चलता है, उसके बाद अपने दाएँ मुड़कर 2 किमी चलता है, और पुनः दाएँ मुड़कर चलता है। अब वह किस दिशा के सम्मुख है?

    • 66% answered correctly
    • Your first attempt
    • Your first attempt
    Re-attempt answer
    Click here to see the answer now
    Reattempt mode is Off. Turn it on from bottom bar

    Your First Attempt Answers

  • Accepted answer is between: 0 and 0
  • हल:

     

    उपरोक्त आकृति में दर्शाए गए अनुसार,

    इसलिए अब व्यक्ति दक्षिण दिशा के सम्मुख है। 

    Question:
    मैराथन दौड़ की दूरी क्या है?
    • 53% answered correctly
    • Your first attempt
    • Your first attempt
    Re-attempt answer
    Click here to see the answer now
    Reattempt mode is Off. Turn it on from bottom bar

    Your First Attempt Answers

  • Accepted answer is between: 0 and 0
  • सही उत्तर 26.2 मील है।

    Key Points

    • एक मैराथन के लिए लंबाई 26.2 मील (42.195 किलोमीटर) है
    • इस घटना की शुरुआत यूनानी सैनिक फेइदिपिड्स के युद्ध में भाग लेने के लिए की गई थी, जिसने मैराथन की लड़ाई से एथेंस तक एक दूत को जीत की सूचना दी थी।
    • मैराथन दौड़ने या रन/वॉक रणनीति के साथ पूरा किया जा सकता है।
    Question:
    फारेनहाइट पैमाने में जल का हिमांक कितना होता है?
    • 72% answered correctly
    • Your first attempt
    • Your first attempt
    Re-attempt answer
    Click here to see the answer now
    Reattempt mode is Off. Turn it on from bottom bar

    Your First Attempt Answers

  • Accepted answer is between: 0 and 0
  • सही उत्‍तर  32°F.है .

    व्याख्या:

    तापमान: यह शरीर की गर्माहट और ठंडक की डिग्री का माप है।
    तापमान का SI मात्रक केल्विन (K) है।
    आमतौर पर इस्तेमाल किए जाने वाले विभिन्न तापमान पैमाने सेल्सियस (C), केल्विन (K), फारेनहाइट (F), और रैंकिन (Ra) हैं।
    सेल्सियस:

    • इसमें जल के हिमांक और क्वथनांक के बीच 100 डिग्री होता है।
    • यहाँ 0° हिमांक को निरूपित करता है और 100° क्वथनांक को दर्शाता है। कई देशों में सेल्सियस का उपयोग दैनिक तापमान को बताने के लिए किया जाता है।

    फारेनहाइट:

    • शुद्ध जल 32o फ़ारेनहाइट पर जम जाता है और 212o फ़ारेनहाइट पर उबलता है।
    • फारेनहाइट पैमाने का प्रस्ताव भौतिक विज्ञानी डैनियल गेब्रियल फारेनहाइट ने 1724 में किया था।
    • यह पैमाना मुख्य रूप से संयुक्त राज्य अमेरिका और कुछ कैरिबियाई देशों में दिन-प्रतिदिन के तापमान को संप्रेषित करने के लिए उपयोग किया जाता है।

    सेल्सियस और फारेनहाइट के बीच संबंध:

    • डिग्री फारेनहाइट (°F) में तापमान T = डिग्री सेल्सियस (°C) में तापमान T गुणा 9/5 + 32:
      • °F=95°C+32

    गणना:

    दिया गया है:
    तापमान (T) = 0°C
    ऊपर दिए गए सूत्र का प्रयोग कीजिये:
    °F=95X°0+32 = 0 + 32 = 32°F

    अतः, यह निष्कर्ष निकाला गया है कि फारेनहाइट पैमाने पर जल 32°F पर जम जाता है।

    Additional Information

    सेल्सियस को केल्विन पैमाने में बदलने के लिए:

    • K = C + 273.15
    • वह तापमान जो सेल्सियस और फारेनहाइट पैमाने में समान माप दिखाता है = -40
    • तापमान जो केल्विन और फारेनहाइट पैमाने में समान माप दिखाता है = 574.25
    Question:
    ‘जय हिन्द’ अभिवादन किसने शुरू किया?
    • 82% answered correctly
    • Your first attempt
    • Your first attempt
    Re-attempt answer
    Click here to see the answer now
    Reattempt mode is Off. Turn it on from bottom bar

    Your First Attempt Answers

  • Accepted answer is between: 0 and 0
  • विकल्प 3 सही है, अर्थात् सुभास चंद्र बोस

    • सुभाष चंद्र बोस:
      • सुभाष चंद्र बोस ने 'जय हिंद' का अभिवादन शुरू किया।
      • नेताजी सुभाष चंद्र बोस द्वारा 22 जून 1939 को भारतीय राष्ट्रीय कांग्रेस के फॉरवर्ड ब्लॉक का गठन किया गया, जिन्होंने 29 अप्रैल को मोहनदास के गांधी द्वारा सत्ता से बाहर किए जाने के बाद भारतीय राष्ट्रीय कांग्रेस के अध्यक्ष पद से इस्तीफा दे दिया था।
      • सुभाष चंद्र बोस और सरदुल सिंह कविश क्रमशः पार्टी के पहले अध्यक्ष और उपाध्यक्ष थे।
      • इसने 20-22 जून, 1940 को नागपुर में अपना पहला सम्मेलन आयोजित किया, जहाँ इसने भारतीय को पूर्ण स्वतंत्रता की मांग की।
      • फारवर्ड ब्लॉक नाम का इसका अपना अखबार था।

    अतिरिक्त तथ्य:

    • पंडित जवाहरलाल नेहरू:
      • पंडित नेहरू भारतीय राजनीति में एक प्रमुख व्यक्ति थे क्योंकि वे स्वतंत्रता से पहले एक स्वतंत्रता सेनानी थे और स्वतंत्रता के बाद भारत के पहले प्रधानमंत्री थे।
      • उनका जन्मदिन (14 नवंबर) को राष्ट्रीय बाल दिवस के रूप में मनाया जाता है क्योंकि वे बच्चों से बहुत प्यार करते थे।
    • मोहनदास करमचन्द गांधी:
      • महात्मा गांधी, मोहनदास करमचंद गांधी के नाम से, (जन्म 2 अक्टूबर, 1869, पोरबंदर, भारत- 30 जनवरी, 1948 को दिल्ली में मृत्यु हो गई, )
      • वह एक वकील, राजनेता, सामाजिक कार्यकर्ता और लेखक थे, जो भारत के ब्रिटिश शासन के खिलाफ राष्ट्रवादी आंदोलन के नेता बने।
    • बाल गंगाधर तिलक:
      • केशव गंगाधर तिलक के रूप में पैदा हुए बाल गंगाधर तिलक (या लोकमान्य तिलक) एक भारतीय राष्ट्रवादी, शिक्षक, समाज सुधारक, वकील और एक स्वतंत्रता कार्यकर्ता थे। वह भारतीय स्वतंत्रता आंदोलन के पहले नेता थे।
      • ब्रिटिश औपनिवेशिक अधिकारियों ने उन्हें "भारतीय अशांति का जनक" कहा। उन्होंने यह भी कहा, "स्वराज मेरा जन्मसिद्ध अधिकार है और मेरे पास होगा।"
    Question:
    यदि एक स्थान पर गुरुत्वाकर्षण के कारण त्वरण 9.8 m/s2 है, तो 40 kg द्रव्यमान वाले एक वस्तु का भार कितना है?
    • 55% answered correctly
    • Your first attempt
    • Your first attempt
    Re-attempt answer
    Click here to see the answer now
    Reattempt mode is Off. Turn it on from bottom bar

    Your First Attempt Answers

  • Accepted answer is between: 0 and 0
  • सही विकल्प 392 N है।

    धारणा:

    गुरुत्वाकर्षण के कारण त्वरण:

    • पृथ्वी द्वारा आकर्षण के गुरुत्वाकर्षण बल के कारण किसी वस्तु द्वारा प्राप्त त्वरण को पृथ्वी द्वारा गुरुत्वाकर्षण के कारण त्वरण कहा जाता है।
    • चूँकि प्रत्येक ग्रह में अलग-अलग द्रव्यमान और त्रिज्या होती है इसलिए गुरुत्वाकर्षण के कारण त्वरण अलग-अलग ग्रह के लिए अलग होगा।

    भार: 

    • किसी वस्तु का भार (w) वस्तु पर गुरुत्वाकर्षण का बल है और इसे गुरुत्वाकर्षण के त्वरण (g) और द्रव्यमान (m) के गुणन के रूप में परिभाषित किया जा सकता है
    • भार एक बल है, भार की SI इकाई न्यूटन है।
    • भार (F) = m x g

    ​चंद्रमा बनाम पृथ्वी:

    • चंद्रमा का द्रव्यमान पृथ्वी के 1/100 गुना है और चंद्रमा की त्रिज्या पृथ्वी के 1/4 गुना है।
    • चूँकि चंद्रमा पर गुरुत्वाकर्षण के कारण त्वरण पृथ्वी के गुरुत्वाकर्षण के कारण त्वरण का छठा भाग है।
    • चंद्रमा की किसी वस्तु का भार = 1/6 × पृथ्वी की सतह पर किसी वस्तु का भार

    दिया हुआ है:

    m = 40 kg g = 9.8 m/s2

    भार = m × g = 40 × 9.8 = 392 N

    Question:
    निम्नलिखित में से कौन सा तत्व बेकिंग सोडा का घटक नहीं है?
    • 55% answered correctly
    • Your first attempt
    • Your first attempt
    Re-attempt answer
    Click here to see the answer now
    Reattempt mode is Off. Turn it on from bottom bar

    Your First Attempt Answers

  • Accepted answer is between: 0 and 0
  • सही उत्तर कैल्शियम है।

    Key Points

    • कैल्शियम बेकिंग सोडा का घटक नहीं है।
    • सोडियम बाइकार्बोनेट बेकिंग सोडा का रासायनिक नाम है।
    • सोडियम बाइकार्बोनेट का दूसरा नाम सोडियम हाइड्रोजन कार्बोनेट है। इसे लोकप्रिय रूप से खाना पकाने का सोडा कहा जाता है।
    • बेकिंग सोडा का रासायनिक सूत्र NaHCO3 है।
    • सोडियम बाइकार्बोनेट ज्वलनशील नहीं है।
    • बेकिंग सोडा प्रकृति में क्षारीय होता है।

    Additional Information

    • सोडियम बाइकार्बोनेट के उपयोग निम्नलिखित हैं: -
      • यह एक एंटासिड के रूप में कार्य करता है जिसका उपयोग पेट के इलाज के लिए किया जाता है।
      • साबुन का झाग बनने के कारण इसका उपयोग अग्निशामक यंत्रों में किया जाता है।
      • यह एक कीटनाशक के रूप में कार्य करता है।
      • यह एक न्यूट्रलाइजर के रूप में एसिड के प्रभाव को बेअसर करता है।
      • पेट में, अम्लता को कम करता है।
    • बेकिंग सोडा का निर्माण सबसे पहले फ्रांसीसी रसायनज्ञ निकोलस लेब्लांक ने किया था।
    • इसे पहली बार 1971 में बनाया गया था।​
    Question:

    बेकारी की समस्या (1) बढ़ती जा रही है(2) शिक्षित वर्ग की (3) दिन प्रतिदिन (4)।

    ऊपर दिए गए वाक्य को क्रमानुसार लगाये।

    • 45% answered correctly
    • Your first attempt
    • Your first attempt
    Re-attempt answer
    Click here to see the answer now
    Reattempt mode is Off. Turn it on from bottom bar

    Your First Attempt Answers

  • Accepted answer is between: 0 and 0
  • सही उत्तर व‍िकल्‍प 4 है। अन्‍य व‍िकल्‍प असंगत हैं। 

    Key Points

    • दिए गये वाक्य का क्रम विकल्प 4 के अनुसार- शिक्षित वर्ग की(3) बेकारी की समस्या(1) दिन प्रतिदिन(4) बढ़ती जा रही है(2)
    • अतः सार्थक वाक्य - "शिक्षित वर्ग की बेकारी की समस्या दिन-प्रतिदिन बढ़ती जा रही है"।
    Question:
    समानार्थी शब्द का चयन कीजिये --नियति---
    • 44% answered correctly
    • Your first attempt
    • Your first attempt
    Re-attempt answer
    Click here to see the answer now
    Reattempt mode is Off. Turn it on from bottom bar

    Your First Attempt Answers

  • Accepted answer is between: 0 and 0
  • नियति शब्द का समानार्थी भाग्य होता है। अन्य विकल्प असंगत है। अतः सही उत्तर विकल्प 4) भाग्य होगा ।

    Key Points

    परिभाषा

    शब्द

    अर्थ

    पर्यायवाची

    समान अर्थवाले शब्दों को 'पर्यायवाची शब्द' या समानार्थक भी कहते है।

    कनक

    1. एक अनाज जिसके आटे की रोटी बनती है
    2. एक पौधा जिसके फलों के बीज बहुत विषैले होते हैं
    3. एक बहुमूल्य पीली धातु जिसके गहने आदि बनते हैं
    4. एक पेड़ जिसमें लाल पुष्प लगते हैं

    कंचन, स्वर्ण, धतूरा


    Additional Information

    शब्द

    पर्यायवाची

    चरित्र

    व्यवहार, आचरण, करनी, शील, सदाचार, आचार

    स्वभाव

    फितरत, प्रकृति, प्रवृत्ति, मनोवृत्ति, मिजाज

    कर्म

    काम, करनी, कार्य

    नियति

    भाग्य, किस्मत, होनी, विधि

    Question:
    बी.सी. सुरेश को किस खेल में योगदान के लिए वर्ष 2022 का ध्यानचंद पुरस्कार (जीवनपर्यंत उपलब्धि) प्राप्त हुआ?
    • 13% answered correctly
    • Your first attempt
    • Your first attempt
    Re-attempt answer
    Click here to see the answer now
    Reattempt mode is Off. Turn it on from bottom bar

    Your First Attempt Answers

  • Accepted answer is between: 0 and 0
  • सही उत्तर कबड्डी है।

    Key Points

    • श्री बी.सी. सुरेश को कबड्डी के खेल में उनके योगदान के लिए ध्यानचंद पुरस्कार मिला
    • खेल और खेलों में आजीवन उपलब्धि के लिए ध्यानचंद पुरस्कार भारत सरकार द्वारा खिलाड़ियों की जीवन भर की उपलब्धियों का सम्मान करने के लिए दिया जाने वाला एक पुरस्कार है।
    • इस पुरस्कार का नाम महान भारतीय हॉकी खिलाड़ी ध्यानचंद के नाम पर रखा गया है, जिन्होंने 1928, 1932 और 1936 में भारत के लिए तीन ओलंपिक स्वर्ण पदक जीते थे
    • यह पुरस्कार 2002 में उन खिलाड़ियों को सम्मानित करने के लिए शुरू किया गया था, जिन्होंने सक्रिय खेल करियर से सेवानिवृत्ति के बाद भी खेल को बढ़ावा देने में योगदान दिया है।
    • श्री बी.सी. सुरेश को 2022 में खेल और खेलों में आजीवन उपलब्धि के लिए ध्यानचंद पुरस्कार मिला।

    Additional Information

    • ध्यानचंद पुरस्कार:
      • ध्यानचंद पुरस्कार भारत में खेलों में आजीवन उपलब्धि के लिए दिया जाने वाला सर्वोच्च पुरस्कार है। आधिकारिक तौर पर, इसे "खेल और खेलों में जीवन भर की उपलब्धि के लिए ध्यानचंद पुरस्कार" भी कहा जाता है।
      • इस पुरस्कार का नाम भारत के महानतम हॉकी खिलाड़ियों में से एक ध्यानचंद के नाम पर रखा गया है। चंद फील्ड हॉकी में अपने तीन ओलंपिक स्वर्ण पदकों के अलावा, अपने असाधारण गोल स्कोरिंग कारनामों के लिए प्रसिद्ध थे, जिसके कारण उन्हें व्यापक रूप से सर्वकालिक महान खिलाड़ी माना जाता था।
      • यह प्रतिष्ठित पुरस्कार 2002 में प्रारम्भ किया गया था। पहले प्राप्तकर्ता शाहूराज बिराजदार (मुक्केबाजी), अशोक दीवान (हॉकी) और अपर्णा घोष (बास्केटबॉल) थे।
      • आमतौर पर, यह पुरस्कार एक वर्ष में तीन से अधिक खिलाड़ियों को नहीं दिया जाता है, हालांकि वर्ष 2003, 2012-2013 और 2018-2020 में अपवाद बनाए गए हैं, जब अधिक प्राप्तकर्ताओं को सम्मानित किया गया था।
      • पुरस्कार में एक प्रतिमा, एक प्रमाण पत्र, एक औपचारिक पोशाक और एक नकद पुरस्कार शामिल है। यह न केवल प्राप्तकर्ताओं को सम्मानित करने के लिए बल्कि पूरे देश में खेलों में उत्कृष्टता की खोज को बढ़ावा देने के लिए भी कार्य करता है।
    Question:
    फीफा विश्व कप 2022 जीतने के लिए अर्जेंटीना ने किस देश को हराया था?
    • 62% answered correctly
    • Your first attempt
    • Your first attempt
    Re-attempt answer
    Click here to see the answer now
    Reattempt mode is Off. Turn it on from bottom bar

    Your First Attempt Answers

  • Accepted answer is between: 0 and 0
  • सही उत्तर फ्रांस है।

    Key Points

    • अर्जेंटीना ने 18 दिसंबर 2022 को 36 साल बाद, अपना तीसरा विश्व कप जीतने के लिए पेनल्टी शूटआउट में फ्रांस को 4-2 से हराया।
    • गोल्डन बॉल का पुरस्कार लियोनेल मेसी को दिया गया।
    • उन्हें यह पुरस्कार 2014 में भी मिला था।
    • आठ गोल के साथ, किलियन एम्बाप्पे को गोल्डन बूट से सम्मानित किया गया।
    • एंजो फर्नांडीज को यंग प्लेयर ऑफ द टूर्नामेंट पुरस्कार से सम्मानित किया गया।
    • जबकि गोलकीपर ऑफ़ टूर्नामेंट का पुरस्कार एमिलियानो मार्टिनेज को दिया गया।

    Additional Information

    • अन्य खेल समाचार:
      • भारतीय तैराक चाहत अरोड़ा ने 14 दिसंबर 2022 को मेलबर्न, ऑस्ट्रेलिया में FINA विश्व तैराकी चैंपियनशिप 2022 में महिलाओं के 100 मीटर ब्रेस्टस्ट्रोक में राष्ट्रीय रिकॉर्ड बनाया।
      • 11 दिसंबर 2022 को बैंकॉक में सीज़न-एंडिंग वर्ल्ड टूर फ़ाइनल में पुरुषों के बैडमिंटन विश्व के नंबर एक विक्टर एक्सेलसेन ने इंडोनेशियाई ऐस एंथनी गिंटिंग को हराकर पुरुष एकल खिताब जीता।
      • भारतीय तैराक शिव श्रीधर ने 13 दिसंबर 2022 को मेलबर्न में शॉर्ट कोर्स वर्ल्ड चैंपियनशिप में पुरुषों की 200 मीटर व्यक्तिगत मेडले हीट में राष्ट्रीय रिकॉर्ड तोड़ा।
      • मनु भाकर ने भोपाल में राष्ट्रीय निशानेबाजी चैंपियनशिप में 10 मीटर पिस्टल जूनियर महिला स्पर्धा में स्वर्ण पदक जीता।
    Question:
    कौन-सा राज्य में 'चिखल कालो' उत्सव मनाया जाता है?
    • 23% answered correctly
    • Your first attempt
    • Your first attempt
    Re-attempt answer
    Click here to see the answer now
    Reattempt mode is Off. Turn it on from bottom bar

    Your First Attempt Answers

  • Accepted answer is between: 0 and 0
  • सही उत्तर गोवा है।

    In News

    • गोवा का यह गांव 'चिखल कालो' उत्सव के लिए पर्यटकों को लुभाता है।

    Key Points

    • उत्तरी गोवा के एक छोटे से गांव मार्सेल ने भगवान कृष्ण के बाल्यव्यस्था का जश्न मनाने वाले पारंपरिक मिट्टी त्योहार "चिखल कालो" के लिए सैकड़ों आगंतुकों को आकर्षित किया।
    • इस कार्यक्रम को देखने के लिए सैकड़ों स्थानीय लोग और पर्यटक गांव में एकत्र हुए, जिसे राज्य पर्यटन विभाग द्वारा मान्यता दी गई है और प्रचारित किया गया है।
    • यह उत्सव मार्सेल के देवकी कृष्ण मंदिर में शुरू होता है और इसमें युवा और बच्चे एक-दूसरे पर कीचड़ फेंकते हैं और भगवान कृष्ण द्वारा एक बच्चे के रूप में खेले गए खेलों की नकल करते हैं।
    Question:
    निम्नलिखित में से कौन सा देश 2022 में ककड़ी और खीरा का सबसे बड़ा निर्यातक बनकर उभरा है?
    • 76% answered correctly
    • Your first attempt
    • Your first attempt
    Re-attempt answer
    Click here to see the answer now
    Reattempt mode is Off. Turn it on from bottom bar

    Your First Attempt Answers

  • Accepted answer is between: 0 and 0
  • सही उत्तर भारत है।

    Key Points

    • भारत दुनिया में ककड़ी और खीरा का सबसे बड़ा निर्यातक बनकर उभरा है।
    • भारत ने अप्रैल-अक्टूबर 2020-21 के दौरान 114 मिलियन अमेरिकी डॉलर के मूल्य के साथ 1,23,000 मीट्रिक टन से अधिक का ककड़ी और खीरा निर्यात किया है।
    • खीरा वर्तमान में संयुक्त राज्य अमेरिका, फ्रांस, जर्मनी, ऑस्ट्रेलिया, स्पेन, दक्षिण कोरिया, कनाडा, जापान, बेल्जियम, रूस और चीन सहित 20 से अधिक देशों को निर्यात किया जाता है।

    Additional Information

    • व्यापार और विकास पर संयुक्त राष्ट्र सम्मेलन (UNCTAD) निवेश रुझान मॉनिटर के अनुसार, 2020 की तुलना में भारत में प्रत्यक्ष विदेशी निवेश (FDI) का प्रवाह 2021 में 26 प्रतिशत गिर गया।
    • व्यापार और विकास पर संयुक्त राष्ट्र सम्मेलन (UNCTAD):
      • मुख्यालय: जिनेवा, स्विट्ज़रलैंड
      • महासचिव: रेबेका ग्रिनस्पैन (जनवरी 2022 तक)
      • स्थापना: 30 दिसंबर 1964
    • जाह्नवी डांगेती ने अमेरिका के अलबामा में कैनेडी स्पेस सेंटर में नासा के अंतर्राष्ट्रीय वायु और अंतरिक्ष कार्यक्रम (iasp NASA) को पूरा कर लिया है, जिससे वह यह उपलब्धि हासिल करने वाली पहली भारतीय बन गई हैं।
    • इंडिया रेटिंग्स एंड रिसर्च (Ind-Ra) ने 2022-23 (FY23) में भारतीय अर्थव्यवस्था की वास्तविक सकल घरेलू उत्पाद (GDP) की विकास दर 7.6 प्रतिशत साल-दर-साल बढ़ने का अनुमान लगाया है।
    • वरिष्ठ नौकरशाह विक्रम देव दत्त को एयर इंडिया लिमिटेड के अध्यक्ष और प्रबंध निदेशक (CMD) के रूप में नियुक्त किया गया है।
    • भारत ने जनवरी 2022 में मुंबई, नवी मुंबई और पुणे में एएफसी महिला फुटबॉल एशियाई कप भारत 2022 की मेजबानी की।
    • भारत की नवदीप कौर ने नेवादा, लास वेगास में प्रतिष्ठित मिसेज वर्ल्ड 2022 पेजेंट में सर्वश्रेष्ठ राष्ट्रीय पोशाक का पुरस्कार जीता है।​
    Question:
    अखिल भारतीय दलित वर्ग कांग्रेस का पहला सत्र 1930 में __________ में आयोजित किया गया था।
    • 43% answered correctly
    • Your first attempt
    • Your first attempt
    Re-attempt answer
    Click here to see the answer now
    Reattempt mode is Off. Turn it on from bottom bar

    Your First Attempt Answers

  • Accepted answer is between: 0 and 0
  • सही उत्तर नागपुर है।

     Key Points

    • नागपुर में दलित वर्ग कांग्रेस में, डॉ. अम्बेडकर ने एक महत्वपूर्ण भाषण की अध्यक्षता की और एक महत्वपूर्ण भाषण दिया जिसमें उन्होंने डोमिनियन स्थिति का समर्थन किया, गांधी के नमक मार्च और सविनय अवज्ञा आंदोलन को अनुचित बताया, लेकिन अकाल और गरीबी के साथ ब्रिटिश औपनिवेशिक कुशासन की भी आलोचना की।
    • दूसरे गोलमेज सम्मेलन में डॉ. बी.आर. अम्बेडकर, जिन्होंने 1930 में दलितों को डिप्रेस्ड क्लास एसोसिएशन में संगठित किया, दलित समुदाय के लिए अलग निर्वाचक मंडल के उनके प्रस्ताव पर महात्मा गांधी के साथ संघर्ष किया।

     Additional Information

    • दलित बौद्ध आंदोलन भारतीय न्यायविद्, अर्थशास्त्री, राजनीतिज्ञ और समाज सुधारक भीमराव रामजी अम्बेडकर से प्रेरित था, जिन्होंने अछूतों (दलितों) के खिलाफ सामाजिक भेदभाव के खिलाफ भी लड़ाई लड़ी थी।
    • अम्बेडकर ने जाति को ध्यान में रखे बिना सभी दुखी व्यक्तियों को दलित के रूप में परिभाषित करके दलित शब्द को लोकप्रिय बनाया।
    • दलित वर्गों के बीच शिक्षा को प्रोत्साहित करने के लिए, उन्होंने 1924 में बंबई में दलित वर्ग संस्थान (बहिष्कृत हितकारिणी सभा) की स्थापना की।
    • सूरत:
      • गुजरात, पश्चिमी भारत के एक राज्य में सूरत शहर शामिल है। सूरत का गुजराती और हिंदी में शाब्दिक अर्थ "सूरत" होता है।
      • यह एक बार एक बड़ा बंदरगाह था और ताप्ती नदी के तट पर स्थित था जहां यह अरब सागर से मिलता था।
      • पश्चिमी भारत के सबसे बड़े शहरी केंद्रों में से एक, यह वर्तमान में दक्षिण गुजरात का वाणिज्यिक और आर्थिक केंद्र है।
      • यह कपड़े और सामान की आपूर्ति के लिए एक महत्वपूर्ण केंद्र है और यहां अच्छी तरह से स्थापित कपड़ा और हीरा उद्योग हैं।
      • यह वह शहर है जहां दुनिया के 90% हीरे काटे और पॉलिश किए जाते हैं।
      • यह आबादी के हिसाब से भारत का आठवां सबसे बड़ा शहर है और कुल मिलाकर नौवां सबसे बड़ा शहरी समूह है।
      • यह 80 मील (किलोमीटर) मुंबई के उत्तर में अहमदाबाद के बाद गुजरात का दूसरा सबसे बड़ा शहर है।
    • दिल्ली:
      • नई दिल्ली, देश की राजधानी, दिल्ली में स्थित है, एक भारतीय शहर और केंद्र शासित प्रदेश, जिसे आधिकारिक तौर पर दिल्ली के राष्ट्रीय राजधानी क्षेत्र (NCT) के रूप में जाना जाता है।
      • दिल्ली पूर्व में उत्तर प्रदेश राज्यों के साथ और शेष दिशाओं में हरियाणा राज्य के साथ सीमाएं साझा करती है क्योंकि विशेष रूप से इसके पश्चिमी या दाहिने किनारे पर यमुना नदी तक फैली हुई है।
      • NCT का आकार 1,484 वर्ग किलोमीटर (573 वर्ग मील) है।
      • NCT की आबादी लगभग 16.8 मिलियन है, जबकि 2011 की जनगणना के अनुसार दिल्ली के शहर की आबादी 11 मिलियन से अधिक थी।
      • राष्ट्रीय राजधानी क्षेत्र (NCR), जिसमें दिल्ली का शहरी समूह और गाजियाबाद, फरीदाबाद, गुड़गांव और नोएडा के उपग्रह शहर शामिल हैं, 28 मिलियन से अधिक लोगों का घर है, जो इसे दुनिया का सबसे बड़ा महानगरीय क्षेत्र बनाता है।
    • कानपुर:
      • भारत के उत्तर प्रदेश राज्य के मध्य-पश्चिमी क्षेत्र में कानपुर का औद्योगिक शहर है।
      • कानपुर, जिसे 1207 में स्थापित किया गया था, ब्रिटिश भारत के सबसे महत्वपूर्ण वाणिज्यिक और सैन्य केंद्रों में से एक के रूप में प्रमुखता से बढ़ा है।
      • कानपुर ने उत्तर प्रदेश के वित्तीय केंद्र के रूप में भी काम किया है।
      • कानपुर, जो गंगा नदी के तट पर स्थित है, भारत की नौवीं सबसे बड़ी शहरी अर्थव्यवस्था और उत्तर भारत का प्रमुख वित्तीय और औद्योगिक केंद्र है।
      • आजकल, यह अपनी औपनिवेशिक वास्तुकला, पार्कों, बगीचों, और उच्च गुणवत्ता वाले चमड़े, प्लास्टिक और कपड़ा उत्पादों के लिए प्रसिद्ध है जो मुख्य रूप से पश्चिम को बेचे जाते हैं।
    Question:
    निम्नलिखित में से किस विटामिन को नियासिन के नाम से जाना जाता है?
    • 59% answered correctly
    • Your first attempt
    • Your first attempt
    Re-attempt answer
    Click here to see the answer now
    Reattempt mode is Off. Turn it on from bottom bar

    Your First Attempt Answers

  • Accepted answer is between: 0 and 0
  • सही उत्तर विकल्प 2 है।

    Key Points

    • विटामिन B3, जिसे नियासिन या निकोटिनिक एसिड भी कहा जाता है, आठ B विटामिनों में से एक है। यह शरीर के उचित कामकाज में महत्वपूर्ण भूमिका निभाता है और विभिन्न चयापचय प्रक्रियाओं में शामिल होता है।
    • यहां विटामिन B3 के बारे में कुछ मुख्य बातें दी गई हैं:
      • कार्य: कार्बोहाइड्रेट, वसा और प्रोटीन को ऊर्जा में बदलने के लिए नियासिन आवश्यक है। 
      • स्रोत: नियासिन पशु और पौधे दोनों स्रोतों से प्राप्त किया जा सकता है। 
      • रूप: नियासिन दो रूपों में मौजूद है: निकोटिनिक एसिड और नियासिनमाइड (निकोटिनमाइड)। 
      • कमी: नियासिन की गंभीर कमी से पेलाग्रा नामक स्थिति हो जाती है।
      • अनुशंसित दैनिक सेवन: नियासिन की अनुशंसित दैनिक सेवन उम्र, लिंग और जीवन स्तर के आधार पर भिन्न होती है। 

    Additional Information

    • विटामिन B1, जिसे थायमिन के रूप में भी जाना जाता है, आठ B-विटामिन में से एक है जो हमारे शरीर के समुचित कार्य को बनाए रखने में आवश्यक भूमिका निभाते हैं।
    • विटामिन B2, जिसे राइबोफ्लेविन के रूप में भी जाना जाता है, बी-कॉम्प्लेक्स समूह से संबंधित पानी में घुलनशील विटामिनों में से एक है। यह समग्र स्वास्थ्य को बनाए रखने में महत्वपूर्ण भूमिका निभाता है और शरीर में विभिन्न चयापचय प्रक्रियाओं में शामिल होता है।
    Question:
    भारतीय संविधान का कौन सा अनुच्छेद भारत को 'राज्य संघ' के रूप में वर्णित करता है?
    • 85% answered correctly
    • Your first attempt
    • Your first attempt
    Re-attempt answer
    Click here to see the answer now
    Reattempt mode is Off. Turn it on from bottom bar

    Your First Attempt Answers

  • Accepted answer is between: 0 and 0
    • संविधान में अनुच्छेद 1 में कहा गया है कि भारत, राज्यों का संघ होगा।
    • भारत के क्षेत्र में राज्य के प्रदेश, केंद्र शासित प्रदेश और अधिग्रहित किये गए कोई भी प्रदेश शामिल होंगे।
    Question:
    'सर्वेन्ट्स ऑफ इंडिया सोसाइटी' के संस्थापक कौन हैं?
    • 63% answered correctly
    • Your first attempt
    • Your first attempt
    Re-attempt answer
    Click here to see the answer now
    Reattempt mode is Off. Turn it on from bottom bar

    Your First Attempt Answers

  • Accepted answer is between: 0 and 0
  • सही उत्तर गोपाल कृष्ण गोखले है।
    Important Points
    • गोपाल कृष्ण गोखले ने 12 जून, 1905 को पुणे, महाराष्ट्र में सर्वेन्ट्स ऑफ़ इंडिया सोसाइटी का गठन किया।
    • यह समाज राजनीतिक गतिविधियों और राजनीतिक संगठन से दूर रहने का विकल्प चुनता है।
    • इस समाज के नेता और सदस्य मुख्य रूप से भारतीय थे।
    • स्वच्छता, स्वास्थ्य देखभाल, शिक्षा को बढ़ावा देने, और भेदभाव, गरीबी, महिलाओं पर अत्याचार और घरेलू शोषण से लड़ने के लिए कई अभियान सर्वेन्ट्स ऑफ़ इंडिया सोसाइटी द्वारा आयोजित किए गए थे।
    Additional Information
    समाज/संगठन  द्वारा स्थापित
    भारतीय स्काउटिंग के संस्थापकों में से एक हृदयनाथ कुंजरू
    सत्यशोधक समाज ज्योतिबा फुले
    बुधि वर्द्धक सभा गोपाल हरि देशमुख
    ब्रह्मो समाज

    राजा राममोहन राय

    Question:
    राज्यसभा के सदस्यों को कौन मनोनीत करता है?
    • 74% answered correctly
    • Your first attempt
    • Your first attempt
    Re-attempt answer
    Click here to see the answer now
    Reattempt mode is Off. Turn it on from bottom bar

    Your First Attempt Answers

  • Accepted answer is between: 0 and 0
  • सही उत्तर राष्ट्रपति है।

    • राज्य सभा एक स्थायी निकाय है और विघटन के अधीन नहीं है।
    • राज्यसभा की अधिकतम संख्या 250 निर्धारित की गई है
      • 238 प्रतिनिधि राज्यों और केंद्र शासित प्रदेशों के  होने चाहिए।
      • 12 प्रतिनिधियों नामांकन राष्ट्रपति द्वारा किया जाता हैं।
    • इसके एक-तिहाई सदस्य हर दूसरे वर्ष सेवामुक्त हो जाते हैं।
    • राज्यसभा के सदस्य के पद की अवधि 6 वर्ष होती है।
    • राज्यसभा के सदस्य के लिए योग्यता:
      • वह भारत का नागरिक होना चाहिए।
      • राज्यसभा के चुनाव के लिए व्यक्ति की आयु 30 वर्ष से कम नहीं होनी चाहिए।
      • उसे संसद द्वारा निर्धारित अन्य योग्यताओं का पालन करना चाहिए।
      • उसे एक संसदीय निर्वाचन क्षेत्र के लिए एक निर्वाचक के रूप में पंजीकृत होना चाहिए।
    Question:
    हमारे शरीर का कौन-सा अंग पेप्सिन का स्राव करता है?
    • 24% answered correctly
    • Your first attempt
    • Your first attempt
    Re-attempt answer
    Click here to see the answer now
    Reattempt mode is Off. Turn it on from bottom bar

    Your First Attempt Answers

  • Accepted answer is between: 0 and 0
  • उत्तर विकल्प 4 है।Key Points

    • हमारे आमाशय में उपस्थित जठर मुख्य कोशिकाएं पेप्सिन का स्राव करती हैं।
      • यह प्रोटीन के पाचन के लिए उत्तरदायी होता है।
    • आमाशय भोजन के पाचन के लिए उत्तरदायी होता है।​
      • यह पाचन में सहायता करने के लिए HCl (हाइड्रोक्लोरिक अम्ल) का स्राव करता है।

    Additional Information

    • यकृत 
      • यह मानव शरीर का सबसे बड़ा अंग है।
      • यह पित्त का स्राव करता है जो प्रोटीन, कार्बोहाइड्रेट और वसा (वसा को छोटे कणों में विघटित करता है) को पचाता है
      • यह अतिरिक्त ग्लूकोज को ग्लाइकोजन (संग्रहित ऊर्जा) में भी परिवर्तित करता है।
      • अतिरिक्त एमीनो अम्ल को अमोनिया में परिवर्तित करता है।
    • अग्न्याशय
      • यह मानव शरीर की दूसरी सबसे बड़ी ग्रंथि है।
      • यह इंसुलिन का स्राव करता है, जिसकी कमी से मधुमेह (डायबिटीज) होता है, और अधिकता से हाइपरग्लेसेमिया होता है।
      • यह कार्बोहाइड्रेट, प्रोटीन और न्यूक्लिक अम्ल का पाचन करता है।
    • थाइरॉयड ग्रंथि
      • यह थाइरॉयड हार्मोन को स्रावित करती है।
      • यह शरीर के उपापचयी, मांसपेशीय और पाचन क्रिया, मस्तिष्क के विकास तथा हड्डियों के अनुरक्षण को बनाए रखने के लिए उत्तरदायी होता है।
    Question:

    निम्नलिखित में से किस समझौते / संधि के बाद, सभी औपचारिक और प्रभावी अधिकार पेशवा को दिए गए थे?

    • 29% answered correctly
    • Your first attempt
    • Your first attempt
    Re-attempt answer
    Click here to see the answer now
    Reattempt mode is Off. Turn it on from bottom bar

    Your First Attempt Answers

  • Accepted answer is between: 0 and 0
    • 1750 में, साहू और उनके पेशवा के बीच एक संवैधानिक समझौते पर हस्ताक्षर किए गए जिसे सांगोला समझौते के रूप में जाना जाता है।

     

    • सांगोला समझौते के अनुसार, सभी प्रभावी और औपचारिक अधिकार पेशवा को दिए गए थे।

    सूरत की संधि-:

    • 1775 में, रघुनाथ राव और ब्रिटिश सेनाओं के बीच इस संधि पर हस्ताक्षर किए गए, जिसके तहत उन्होंने अंग्रेजों को मराठाओं पर सीधा हमला करने के लिए आमंत्रित किया।

     

    सालबाई की संधि-:

    • इस संधि के अनुसार, ब्रिटिश और मराठा बल भावी 20 वर्षों तक एक-दूसरे के विरुद्ध युद्ध की घोषणा नहीं करने के लिए सहमत हुए थे ।

    राजपुर घाट की संधि-:

    • इस संधि पर यशवंत राव होलकर ने हस्ताक्षर किए थे, इस संधि के अनुसार प्रमुख मराठा क्षेत्रों को ब्रिटिश बलों को सौंप दिया गया था।
    Question:
    बीजक, किस संत की रचनाओ का संकलन है?
    • 66% answered correctly
    • Your first attempt
    • Your first attempt
    Re-attempt answer
    Click here to see the answer now
    Reattempt mode is Off. Turn it on from bottom bar

    Your First Attempt Answers

  • Accepted answer is between: 0 and 0
    • "बीजक" के रचनाकार "कबीरदास" है अतः उक्त विकल्पों में से कबीरदास विकल्प (2) सही है तथा अन्य विकल्प असंगत है।
    • कबीर की रचनाओं का संकलन -- उनके शिष्य धर्मदास ने किया है।

    Key Points

    • कबीर की रचनाओं का संकलन "बीजक" कहलाता है।
    • इस कृति को कबीर पंथ की पवित्र पुस्तक मानी जाती है।
    • बीजक के तीन भाग है:-
      • साखी 
      • सबद 
      • रमैनी

    Additional Information

    बीजक के भाग

    संबंधित

    रमैनी

    • मूलबीह्ज पहला प्रकरण है। 
    • इसमें 'रमैनी' नामक चौरासी पद्ध हैं।

    शब्द

    • यह दूसरा प्रकरण है। 
    • इसमें 'शब्द' नामक 115 पद्ध हैं।
    Question:
    अनुच्छेद 226 के तहत उच्च न्यायालय के पास किससे संबंधित प्रवर्तन हेतु अधिकार-क्षेत्र है। 
    • 50% answered correctly
    • Your first attempt
    • Your first attempt
    Re-attempt answer
    Click here to see the answer now
    Reattempt mode is Off. Turn it on from bottom bar

    Your First Attempt Answers

  • Accepted answer is between: 0 and 0
  • सही उत्तर मौलिक अधिकार है।

    Key Points

    • मौलिक अधिकार को लागू करने हेतु निर्देश, आदेश या याचिका जारी करने के लिए उच्च न्यायालय के पास अनुच्छेद 226 के तहत अधिकार-क्षेत्र है।
    • एक उच्च न्यायालय को मौलिक अधिकार के प्रवर्तन संबंधी निर्देश, आदेश या याचिका जारी करने का अधिकार है और किसी भी अन्य उद्देश्य हेतु इनमे शामिल हैं:
      • बन्दी प्रत्यक्षीकरण
      • परमादेश
      • प्रतिषेध
      • अधिकार-पृच्छा
      • उत्प्रेषण लेख
    • वाक्यांश 'किसी अन्य उद्देश्य के लिए' एक सामान्य कानूनी अधिकार के प्रवर्तन को संदर्भित करता है।
    • उच्च न्यायालय किसी भी व्यक्ति, प्राधिकरण और सरकार को न केवल अपने क्षेत्रीय अधिकार-क्षेत्र के अंतरगर्त बल्कि अपने क्षेत्रीय-अधिकार क्षेत्र के बाहर भी याचिका जारी कर सकता है यदि कार्रवाई का कारण उसके क्षेत्रीय अधिकार क्षेत्र में उत्पन्न होता है।

    Important Pointsकुल पांच याचिका/रिट का विवरण:

    रिट विवरण

    बन्दी प्रत्यक्षीकरण

    बंदी प्रत्यक्षीकरण याचिका का प्रयोग किसी कैदी या अन्य बंदी को अदालत के समक्ष प्रस्तुत करने हेतु किया जाता है ताकि यह निर्धारित किया जा सके कि व्यक्ति का कारावास या हिरासत वैध है या नहीं।

    परमादेश

    यह एक न्यायिक याचिका है जो एक निचली अदालत को आदेश के रूप में जारी की जाती है या किसी व्यक्ति को सार्वजनिक या वैधानिक कर्तव्य करने का आदेश देती है।

    प्रतिषेध

    यह याचिका अक्सर एक उच्च न्यायालय द्वारा निचली अदालत को जारी की जाती है, जो उसे किसी ऐसे मामले में अधिकारों के प्रयोग पर रोक लगाती या निर्देश देती है जो उसके अधिकार-क्षेत्र के अंतरगर्त नहीं आता।

    अधिकार पृच्छा

    यह याचिका या कानूनी कार्रवाई है जिसके लिए किसी व्यक्ति को यह दिखाने की आवश्यकता होती है कि किसी कार्यालय या मताधिकार का क्या वारंट है, दावा किया गया है या प्रयोग किया गया है।

    उत्प्रेषण लेख

    याचिका जिसके द्वारा अपीलीय अदालत अपने विवेक से किसी मामले की समीक्षा करने का निर्णय लेती है।
    Question:

    एक कार 40 किमी/घंटा की गति से 40 मिनट के लिए, 45 किमी/घंटा की गति से 50 मिनट के लिए और 60 किमी/घंटा की गति से अगले 90 मिनट के लिए यात्रा करती है। पूरी यात्रा में कार की औसत गति (किमी/घंटा में) कितनी है?

    • 38% answered correctly
    • Your first attempt
    • Your first attempt
    Re-attempt answer
    Click here to see the answer now
    Reattempt mode is Off. Turn it on from bottom bar

    Your First Attempt Answers

  • Accepted answer is between: 0 and 0
  • दिया गया है:

    एक कार 40 किमी/घंटा की गति से 40 मिनट के लिए, 45 किमी/घंटा की गति से 50 मिनट के लिए और 60 किमी/घंटा की गति से अगले 90 मिनट के लिए यात्रा करती है।

    प्रयुक्त अवधारणा:

    औसत गति = कुल दूरी/कुल समय

    गणना:

    40 मिनट = 2/3 घंटे

    50 मिनट = 5/6 घंटे

    90 मिनट = 3/2 घंटे

    यात्रा की कुल दूरी = 40 × (2/3) + 45 × (5/6) + 60 × (3/2)

    ⇒ 80/3 + 75/2 + 90

    ⇒ (160 + 225 + 540)/6

    ⇒ 925/6 किमी

    कुल समय = 2/3 + 5/6 + 3/2

    ⇒ (4 + 5 + 9)/6

    ⇒ 18/6 = 3 घंटे

    औसत गति = (925/6)/3

    ⇒ 925/18

    ⇒ 51718 किमी/घंटा

    ∴ पूरी यात्रा में कार की औसत गति (किमी/घंटा में) 51718 है।

    Question:

    Direction: Select the appropriate synonym for the underlined word.

    John says, “l am serious.” 

    • 39% answered correctly
    • Your first attempt
    • Your first attempt
    Re-attempt answer
    Click here to see the answer now
    Reattempt mode is Off. Turn it on from bottom bar

    Your First Attempt Answers

  • Accepted answer is between: 0 and 0
  • The correct answer is "severe."

     Key Points

    • "Serious" is often used to describe a situation or tone and it means demanding careful consideration or application. (गंभीरता से (न कि हलके से) लिया जाने योग्‍य)
      • ExampleDon’t laugh, it’s a serious matter.
    • "Severe" is more commonly used to describe a physical or emotional state and it means very great or intense. (बहुत कठिन, कष्‍टप्रद या गंभीर)
      • Example: There were severe financial difficulties.
    • Synonym for "serious" is "severe."
    • The correct answer is Option 2.
       

    Additional Information

    • Option 1: "Safe" means free from harm or danger. It is not a synonym for "serious". (जिसे कोई ख़तरा न हो, जिसे क्षति न पहुँचाई जा सके; सुरक्षित)
    • Option 3: "Comical" means amusing or funny. It is an antonym of "serious". (हास्‍यकर; विनोदक)
    • Option 4: "Light" means not heavy or not serious. It is an antonym of "serious". (प्रकाश, रोशनी)
    Question:
    निम्नलिखित में से कौन 'जगत कवक' से संबंधित नहीं है?
    • 37% answered correctly
    • Your first attempt
    • Your first attempt
    Re-attempt answer
    Click here to see the answer now
    Reattempt mode is Off. Turn it on from bottom bar

    Your First Attempt Answers

  • Accepted answer is between: 0 and 0
  • सही उत्तर यूग्लीना है

    Key Points

    • यूग्लीना प्रोटिस्टा जगत से संबंधित है।
    • यूग्लीना एक एकल-कोशिका वाला जीव है जिसमें एक सुपरिभाषित केन्द्रक होता है।
    • यह चलन के लिए फ्लैगेला का उपयोग करता है, ये सभी विशेषताएं प्रोटिस्टा जगत से संबंधित जीवों के समान हैं

    Additional Information 

    • सजीवों को पांच अलग-अलग जगत में रखा गया है:
      • मोनेरा
        • प्रोकैरियोटिक जीवों को रखा गया है।
        • वे एक आदिम प्रकार के जीव हैं, कुछ को परपोषी कोशिका पर निर्भर होते हैं, कुछ सहजीवी संबंध को प्रदर्शित करते हैं।
        • उन्हें प्रोकैरियोट्स कहा जाता है क्योंकि उनका केन्द्रक कोशिका द्रव्य में बिना परमाणु झिल्ली के उपस्थित होता है।
      • प्रॉटेस्टा
        • यह जगत प्रोकैरियोट्स जगत के बीच एक जोड़ने वाली कड़ी के रूप में कार्य करता है।
        • ज्यादातर जलीय और समुद्र, मीठे जल और नम मिट्टी में पाए जाते हैं।
      • कवक
        • इस जगत में ऐसी प्रजातियां शामिल हैं जो हेटरोट्रॉफ़िक, बीजाणु-निर्माण, गैर-संवहनी, एकक्लोरोफिलस और यूकेरियोटिक जीव हैं।
        • उनकी कोशिका भित्ति काइटिन या कवक सेलुलोज से बनी होती है और ग्लाइकोजन एक आरक्षित खाद्य सामग्री है।
        • कवकों के अध्ययन को माइकोलॉजी कहते हैं।
        • उदाहरण: एस्परगिलस, म्यूकोर, राइजोपस।
      • पादप
        • यह यूकेरियोटिक बहुकोशिकीय जीवित जीव है जो प्रकाश संश्लेषण प्रक्रिया को करते हैं।
        • पारिस्थितिकी तंत्र में निर्माता के रूप में कार्य करते हैं
        • इनका शरीर जड़ों, टहनियों और पत्तियों में बंटा होता है।
        • उन्हें क्रिप्टो ग्राम (फूल और बीज-असर वाले पौधे) और फ़ैनरोगैमे (बीज और यौन अंगों वाला एक पौधा) में विभाजित किया गया है।
      • जंतु
        • जंतु जगत सभी जंतुओं का गठन करता है।
        • पांच जगत में सबसे बड़ा जगत जंतु जगत है।
        • जंतु बहुकोशिकीय यूकेरियोट्स हैं।
        • हालांकि, पादपों की तरह, उनमें क्लोरोफिल या कोशिका भित्ति नहीं होती है।
    Question:
    निम्नलिखित में से हिंद महासागर के सबसे गहराई वाले स्थान का क्या नाम है?
    • 28% answered correctly
    • Your first attempt
    • Your first attempt
    Re-attempt answer
    Click here to see the answer now
    Reattempt mode is Off. Turn it on from bottom bar

    Your First Attempt Answers

  • Accepted answer is between: 0 and 0
  • सही उत्तर सुन्दा गर्त है।

    • हिंद महासागर के सबसे गहराई वाले स्थान का क्या नाम सुन्दा गर्त है; जिसे जावा गर्त के रूप में भी जाना जाता है।

    Key Points

    • यह हिंद महासागर में जावा के पास स्थित है। इसकी अधिकतम गहराई 7290 मी है और चौड़ाई 3200 किमी है।
    • मारियाना गर्त पृथ्वी पर सबसे गहरी समुद्री खाई है।
    • यह प्रशांत महासागर में स्थित है।
    • अटलांटिक महासागर में सबसे गहरा बिंदु पोर्तो रिको गर्त की गहराई 8376 मीटर है
    • प्यूर्टो रिको गर्त के बाद अटलांटिक महासागर का दूसरा सबसे गहरा बिंदु दक्षिण सैंडविच गर्त है।
    • यह दक्षिणी अटलांटिक महासागर में सबसे गहरा बिंदू है।
    Question:
    ऐक्रेलिक को कृत्रिम ऊन या ______ के रूप में जाना जाता है।
    • 40% answered correctly
    • Your first attempt
    • Your first attempt
    Re-attempt answer
    Click here to see the answer now
    Reattempt mode is Off. Turn it on from bottom bar

    Your First Attempt Answers

  • Accepted answer is between: 0 and 0
  • सही उत्तर संश्लेषित ऊन है।

    Key Points

    • ऐक्रेलिक ऐक्रेलिक रेशे को संदर्भित करता है जो पॉलिमर PAN (पॉलीएक्रिलोनिट्राइल) (C3H3N)n से बने संश्लेषित रेशे हैं।
    • इसे संश्लेषित ऊन भी कहा जाता है क्योंकि यह ऊन के रेशों जैसा दिखता है।

    Additional Information

    • पॉलिएस्टर पॉलिमर का एक समूह है जिसमें एस्टर (C10H8O4)n) एक कार्यात्मक समूह के रूप में होता है।
    • पॉलिएस्टर प्राकृतिक और संश्लेषित दोनों हो सकता है। इसका इस्तेमाल ज्यादातर कपड़ों में किया जाता है।
    • नायलॉन संश्लेषित बहुलक के एक समूह को संदर्भित करता है जिसमें पॉलीएमाइड (CO-NH) होता है। यह रेशम जैसा दिखता है और आमतौर पर पेट्रोलियम से बनाया जाता है।
    • PVC विनाइल क्लोराइड या क्लोरोएथीन (H2C=CHClके एकलक के साथ सबसे व्यापक रूप से इस्तेमाल किए जाने वाले संश्लेषित बहुलक में से एक है। इसका उपयोग प्लास्टिक पाइप और अन्य सामान बनाने में किया जाता है।
    Question:

    निम्नलिखित में से कौन से युग्म सुमेलित हैं?

    अम्ल उपयोग
    1. ऑक्सालिक अम्ल चमड़े का विरंजन
    2. एसिटिक अम्ल खाद्य पदार्थों का प्रसंस्करण
    3. साइट्रिक अम्ल धातु सफाई 
    4. बेंजोइक अम्ल संचय बैटरी

    नीचे दिए गए कूट का प्रयोग कर सही उत्तर चुनिए।

    • 25% answered correctly
    • Your first attempt
    • Your first attempt
    Re-attempt answer
    Click here to see the answer now
    Reattempt mode is Off. Turn it on from bottom bar

    Your First Attempt Answers

  • Accepted answer is between: 0 and 0
  • सही उत्तर 1, 2 और 3 है। 

    Key Pointsऑक्सालिक अम्ल

    • ऑक्सालिक अम्ल, जिसे एथेनेडियोइक अम्ल भी कहा जाता है, कार्बोक्जिलिक अम्ल के परिवार से संबंधित रंगहीन, क्रिस्टलीय, विषाक्त कार्बनिक यौगिक है।
    • विरंजन, धातु पॉलिश, दाग हटाने और मॉर्डेंट (रंगबंधक) के रूप में ऑक्सालिक अम्ल के कई कार्य हैं।
    • इसका उपयोग पुआल टोपी, चमड़ा और लकड़ी के लिए विरंजक एजेंट के रूप में किया जाता है। अत: युग्म 1 सही सुमेलित है।
    • ऑक्सालिक अम्ल धातु के क्साइड के साथ-साथ जंग या स्याही के दाग को हटाने के लिए एक अपचायक एजेंट के रूप में कार्य करता है।
    • ऑक्सालिक अम्ल का सूत्र है (C2H2O4); इसका सामान्य रूप क्रिस्टलीय हाइड्रेट (COOH)2·2H2O का होता है।

    एसीटिक अम्ल

    • एसिटिक अम्ल को एथेनोइक अम्ल, एथिलिक अम्ल, सिरका अम्ल और मीथेन कार्बोक्जिलिक अम्ल के रूप में भी जाना जाता है; इसका रासायनिक सूत्र CH3COOH है। 
    • एसिटिक अम्ल का उपयोग खाद्य उत्पादों की अम्लता को बढ़ाने (और pH कम करने) के लिए किया जा सकता है और साथ ही उत्पाद को नमक और सिरका चिप्स जैसे अम्ल स्वाद देकर ऑर्गेनोलेप्टिक (इंद्रिय ग्राही) गुणवत्ता में सुधार किया जा सकता है। अत: युग्म 2 सही सुमेलित है।
    • एसिटिक अम्ल भी एक लोकप्रिय परिरक्षक है क्योंकि यह ड्रेसिंग, सॉस, पनीर और अचार में जीवाणु के विकास को रोकता है।

    साइट्रिक अम्ल

    • साइट्रिक अम्ल को इसका नाम इसलिए मिला क्योंकि यह पहली बार नींबू, संतरे और नीबू जैसे खट्टे फलों में खोजा गया था।
    • यह अम्ल इन फलों को उनका खट्टा स्वाद देता है।
    • धातुओं को साफ करने के लिए साइट्रिक अम्ल का इस्तेमाल कई तरीकों से किया जाता है। अत: युग्म 3 सही सुमेलित है।

    बेंज़ोइक अम्ल

    • बेंज़ोइक अम्ल (BAभोजन और पेय पदार्थों में, विशेष रूप से कार्बोनेटेड पेय पदार्थों में, आमतौर पर इस्तेमाल किया जाने वाला रोगाणुरोधी परिरक्षक है क्योंकि यह पीएच 2.5–4.0 पर अपनी सबसे मजबूत जीवाणुरोधी गतिविधि प्रस्तुत करता है।अत: युग्म 4 सुमेलित नहीं है।
    • बेंज़ोइक अम्ल का बैक्टीरिया और यीस्ट के प्रसार पर निरोधात्मक प्रभाव पड़ता है, जो भोजन के खराब होने का एक प्रमुख कारण है।
    Question:
    भारतीय संविधान का अनुच्छेद 17 निम्नलिखित में से किससे संबंधित है?
    • 83% answered correctly
    • Your first attempt
    • Your first attempt
    Re-attempt answer
    Click here to see the answer now
    Reattempt mode is Off. Turn it on from bottom bar

    Your First Attempt Answers

  • Accepted answer is between: 0 and 0
  • सही उत्तर अस्पृश्यता का उन्मूलन है।

    • भारतीय संविधान का अनुच्छेद 17 अस्पृश्यता के उन्मूलन से संबंधित है।

    Key Points

    • अनुच्छेद 17 अस्पृश्यता का उन्मूलन सुनिश्चित करता है।
    • मौलिक अधिकार:​
      • मौलिक अधिकारों का वर्णन भारतीय संविधान के अनुच्छेद 12-35, भाग III में किया गया है।
    • संपत्ति के अधिकार (अनुच्छेद 31) को मौलिक अधिकारों की सूची से हटा दिया गया था।​
      • 44वें संशोधन अधिनियम, 1978 द्वारा इसे संविधान के भाग XII में अधिनियम 300A के तहत एक सामान्य संवैधानिक अधिकार बना दिया गया है।

    Additional Information

    • अनुच्छेद 29 अल्पसंख्यकों के हितों के संरक्षण से संबंधित है।
    • अनुच्छेद 16 सार्वजनिक रोजगार के मामलों में अवसर की समानता से संबंधित है।
    • अनुच्छेद 10 नागरिकता के अधिकारों की निरंतरता से संबंधित है।
    Question:
    निम्नलिखित में से कौन अलवार महिला संत है?
    • 32% answered correctly
    • Your first attempt
    • Your first attempt
    Re-attempt answer
    Click here to see the answer now
    Reattempt mode is Off. Turn it on from bottom bar

    Your First Attempt Answers

  • Accepted answer is between: 0 and 0
  • सही उत्तर आंडाल है।  
    Key Points

    आंडाल, एक महिला अलवार भक्ति संत थीं।

    • दक्षिण भारत के 12 आलवार संतों में आंडाल एकमात्र महिला अलवार थीं। 
    • तमिल कृतियों जैसे थिरुप्पावई और नाचियार तिरुमोझी में उनकी महान रचनाएँ हैं।
    • वह भगवान विष्णु के प्रति अपनी अटूट भक्ति के लिए जानी जाती हैं।
    Question:
    18 V की बैटरी पर कार्य करने वाला एक प्रकाश बल्ब 3 A की धारा लेता है। बल्ब का प्रतिरोध क्या होगा?
    • 86% answered correctly
    • Your first attempt
    • Your first attempt
    Re-attempt answer
    Click here to see the answer now
    Reattempt mode is Off. Turn it on from bottom bar

    Your First Attempt Answers

  • Accepted answer is between: 0 and 0
  • सही उत्तर 6 Ω है।

    Key Points

    बल्ब के प्रतिरोध की गणना करने के लिए हम ओम के नियम का उपयोग कर सकते हैं:

    ओम का नियम कहता है कि दो बिंदुओं के बीच एक सुचालक के माध्यम से धारा (I) दो बिंदुओं पर वोल्टता (V) के समानुपाती होती है और उनके बीच के प्रतिरोध (R) के व्युत्क्रमानुपाती होती है:

    I = V / R

    R = V / I

    जहाँ:
    R = ओम में प्रतिरोध (Ω)
    V = वोल्ट में वोल्टता (V)
    I = ऐम्पियर में धारा (A)

    यह देखते हुए कि वोल्टता (V), 18 है और धारा (I) 3 है, हम प्रतिरोध (R) ज्ञात करने के लिए इन मानों को समीकरण में प्रतिस्थापित कर सकते हैं:

    R = V / I

    ⇒ 18 V/ 3 A= 6 Ω

    अतः बल्ब का प्रतिरोध 6 ओम (Ω) है।

    Question:
    _______ निम्न सूचीबद्ध अम्लों में दुर्बलतम है।
    • 28% answered correctly
    • Your first attempt
    • Your first attempt
    Re-attempt answer
    Click here to see the answer now
    Reattempt mode is Off. Turn it on from bottom bar

    Your First Attempt Answers

  • Accepted answer is between: 0 and 0
  • सही उत्तर एसिटिक अम्ल है।

    Key Points

    • क्योंकि एसिटिक अम्ल को पानी में विलय करने पर यह अपने घटक आयनों में आंशिक रूप से अलग हो जाता है इसलिए एसिटिक अम्ल एक कमजोर अम्ल है।
    • यह सर्वविदित है कि यह कमजोर अम्ल पानी के साथ मिश्रित हो सकता है। एक जलीय विलयन में, एसिटिक अम्ल नामक अम्ल थोड़ा-सा आयनित होता है।
    • विशेष रूप से लोकप्रिय कमजोर अम्ल एसिटिक अम्ल होता है, जो सिरके में मौजूद होता है।
    • एसिटिक अम्ल, जिसे आमतौर पर एथेनॉलिक अम्ल के रूप में जाना जाता है, एक तरल कार्बनिक रसायन है जो रंगहीन होता है।
    • विलयन में आयनित किए गए मूल अम्ल की मात्रा को अम्ल आयनीकरण द्वारा दर्शाया जाता है।
    • परिणामस्वरूप, Ka का संख्यात्मक मान अम्ल की ताकत का प्रतिनिधित्व करता है। अपेक्षाकृत कम Ka मान वाले कमजोर अम्ल की तुलना में मजबूत अम्ल में काफी अधिक Ka मान वाले कमजोर अम्ल शामिल होते हैं।

    Important Points

    • फ्लोरीन युक्त रासायनिक यौगिकों में हाइड्रोजन फ्लोराइड शामिल है।
    • यह एक रंगहीन वाष्प के रूप में, उबलते तरल के रूप में या पानी में घुलने वाले पदार्थ के रूप में मौजूद हो सकता है।
    • हाइड्रोफ्लोरिक अम्ल वह है जिसे हाइड्रोजन फ्लोराइड के पानी में घुलने पर जाना जाता है।
    • अकार्बनिक पदार्थ नाइट्रिक अम्ल का सूत्र HNO3 होता है।
    • यह खनिज अम्ल अत्यंत संक्षारक है।
    • हाइड्रोक्लोरिक अम्ल (HCl) के लिए कई औद्योगिक उपयोगों में से एक जब इसे कार्बनिक क्षार के साथ जोड़ा जाता है एक रंगहीन, शक्तिशाली खनिज अम्ल हाइड्रोक्लोराइड लवण का निर्माण होता है।
    Question:
    निम्नलिखित में से कौन लोकसभा के अध्यक्ष को 'अस्थायी' नियुक्त करता है?
    • 66% answered correctly
    • Your first attempt
    • Your first attempt
    Re-attempt answer
    Click here to see the answer now
    Reattempt mode is Off. Turn it on from bottom bar

    Your First Attempt Answers

  • Accepted answer is between: 0 and 0
  • अस्थायी अध्यक्षसही उत्तर राष्ट्रपति है।

    Key Points 

    • राष्ट्रपति लोकसभा के एक सदस्य को अस्थायी अध्यक्ष के रूप में नियुक्त करता है।
    • नव-निर्वाचित लोकसभा के सबसे वरिष्ठ सदस्यों में से एक को आम तौर पर अस्थायी अध्यक्ष के रूप में नियुक्त किया जाता है
    • राष्ट्रपति स्वयं लोकसभा के अस्थायी अध्यक्ष को शपथ दिलाते हैं।
    • अस्थायी अध्यक्ष अस्थायी वक्ता होता है जो आम चुनाव के बाद संसद के निचले सदन की पहली बैठक की अध्यक्षता करने के अलावा उस बैठक की अध्यक्षता करता है जिसमें अध्यक्ष और उपाध्यक्ष होते हैं।
    • जैसा कि संविधान द्वारा प्रदान किया गया है, अंतिम लोकसभा का अध्यक्ष नव-निर्वाचित लोकसभा की पहली बैठक से ठीक पहले अपना पद खाली कर देता है।

    Additional Information 

    • संविधान के अनुच्छेद 94 के दूसरे प्रावधान के अनुसार, नई लोकसभा की पहली बैठक से ठीक पहले अध्यक्ष का पद खाली हो जाता है।
    • उस मामले में विधायी अनुभाग द्वारा तैयार वरिष्ठ लोकसभा सदस्यों की सूची संसदीय कार्य मंत्री को प्रस्तुत की जाती है, जो एक अस्थायी अध्यक्ष का चयन करता है और राष्ट्रपति द्वारा अनुमोदित किया जाता है।
    • एक राष्ट्रपति समर्थक अस्थायी या वक्ता अस्थायी अध्यक्ष एक विधायी निकाय का संवैधानिक रूप से मान्यता प्राप्त अधिकारी है जो सामान्य पीठासीन अधिकारी की अनुपस्थिति में कक्ष की अध्यक्षता करता है।
    अस्थायी अध्यक्ष
    Question:
    मानव कान के उस हिस्से को क्‍या कहते हैं, जो श्रव्य ध्वनि तरंगों से संबद्ध दाब में होने वाले परिवर्तनों को वैद्युत संकेतों में परिवर्तित करता है? 
    • 35% answered correctly
    • Your first attempt
    • Your first attempt
    Re-attempt answer
    Click here to see the answer now
    Reattempt mode is Off. Turn it on from bottom bar

    Your First Attempt Answers

  • Accepted answer is between: 0 and 0
  • सहीउत्तर कर्णावर्त (कॉक्लिया) है

    Key Points

    कर्णावर्त (कॉक्लिया):

    • कान का कर्णावर्त वायु (ध्वनि) में दाब भिन्नता को विद्युत संकेतों में परिवर्तित करता है।
    • ये संकेत श्रवण तंत्रिकाओं के माध्यम से मस्तिष्क तक जाते हैं, जिसके कारण हम उन्हें सुन पाते हैं।
    • कर्णावर्त लेबरिंथ का कुंडलित भाग है।
    • कर्णावर्त आंतरिक कान में उपस्थित मौजूद श्रवण अंग है। कर्णावर्त में श्रवण ग्राही उपस्थित होते हैं।

    कर्णावर्त संरचना:

    • कर्णावर्त, रीस्नर की झिल्ली और आधारीय झिल्ली की कुंडलित झिल्ली परिलसिका से भरी बोनी लेबरिंथ को तीन नलिकाओं में अलग करती है।
      • स्केला वेस्टिबुली- ऊपरी
      • स्केला मीडिया- मध्य
      • स्केला टिम्पनी- निचला

    कर्णावर्त का कार्य:

    • कर्णावर्त में श्रवण ग्राही उपस्थित होते हैं।
    • ध्वनि कंपन से उत्पन्न दाब तरंगें अंतर्कर्णोदक (एंडोलिम्फ) में स्थानांतरित हो जाती हैं और इससे आधारीय झिल्ली में गति होती है।
    • टेक्टोरियल झिल्ली के मूल रोम की कोशिकाओं के झुकने के परिणामस्वरूप आधार पर मौजूद अभिवाही न्यूरॉन में तंत्रिका आवेगों का निर्माण होता है जो रोम की कोशिकाओं के निकट होते हैं।
    • इस प्रकार उत्पन्न क्रिया क्षमता श्रवण तंत्रिकाओं के माध्यम से मस्तिष्क तक जाती है।
    • मस्तिष्क विद्युत संकेत को एकीकृत करता है और हम ध्वनि को पहचानते हैं।
    Question:
    _______ एक साधारण उपकरण है जिसका उपयोग या तो विद्युत परिपथ को वियोजित करने या इसे पूर्ण करने के लिए किया जाता है।
    • 51% answered correctly
    • Your first attempt
    • Your first attempt
    Re-attempt answer
    Click here to see the answer now
    Reattempt mode is Off. Turn it on from bottom bar

    Your First Attempt Answers

  • Accepted answer is between: 0 and 0
  • सही उत्तर स्विच है।

    Key Points

    • स्विच एक साधारण उपकरण है जिसका उपयोग या तो विद्युत परिपथ को वियोजित करने या इसे पूर्ण करने के लिए किया जाता है।
    • प्रतिरोधक एक निष्क्रिय विद्युत घटक है जो विद्युत धारा के प्रवाह का प्रतिरोध करता है।
    • ट्रांजिस्टर एक अर्धचालक उपकरण है जो प्रवर्धक और स्विच दोनों के रूप में कार्य करता है।
    • फ्यूज एक सुरक्षा विद्युत उपकरण है जो विद्युत धारा के बहुत अधिक होने पर इसे परिपथ से हटा देता है।

    Additional Information

    • ट्रांजिस्टर:
      • ट्रांजिस्टर अर्धचालक उपकरण होते हैं, जो आमतौर पर प्रवर्धक या विद्युत नियंत्रित स्विच के रूप में उपयोग किए जाते हैं। 
      • ट्रांजिस्टर आधारभूत निर्माण खंड हैं जो कंप्यूटर, मोबाइल फोन और अन्य सभी आधुनिक विद्युतीय परिपथ के संचालन को नियंत्रित करते हैं। 
      • सबसे महत्वपूर्ण ट्रांजिस्टर कार्य एक वर्तमान प्रवर्धक के रूप में और एक स्विच के रूप में होता है। 
      • ट्रांजिस्टर अक्सर प्रवर्धकों में विद्युत धारा प्रवर्धकों के रूप में  उपयोग किए जाते हैं जबकि ट्रांजिस्टर स्विच के रूप में अक्सर स्वचालित लाइट सर्किट में पाए जाते हैं। 
      • ट्रांजिस्टर के कार्य परिवर्तनीय प्रतिरोधक और स्विचिंग डिवाइस हैं।
    • संधारित्र:
      • संधारित्र एक ऐसा उपकरण है जो विद्युत ऊर्जा को विद्युत क्षेत्र में संग्रहित करता है।
      • यह दो टर्मिनलों वाला एक निष्क्रिय विद्युतीय घटक है।
      • संधारित्र के प्रभाव को धारिता के रूप में जाना जाता है।
      • धारिता: धारिता संधारित्र की आवेश संग्रहित करने की क्षमता है।
      • दो चालकों को एक विद्युतरोधी (परावैद्युत) द्वारा पृथक किया जाता है और जब एक विद्युत क्षेत्र आरोपित किया जाता है, तो उसमें विद्युत ऊर्जा एक आवेश के रूप में संग्रहित हो जाती है।
        • संधारित्र की धारिता (C): किसी चालक की धारिता उसके विभव (V) में वृद्धि से आवेश (Q) का अनुपात होता है, अर्थात
        • C = Q/V
    • प्रतिरोधक:
      • प्रतिरोधक एक निष्क्रिय तत्व होता है।
      • इसे 'R' द्वारा दर्शाया जाता है और इसे ओम (Ω) में मापा जाता है।
      • प्रतिरोधों का उपयोग धारा प्रवाह को कम करने, सिग्नल के स्तर को समायोजित करने, वोल्टेज को विभाजित करने के लिए किया जाता है।
    Question:
    _______ एक साधारण उपकरण है जिसका उपयोग या तो विद्युत परिपथ को वियोजित करने या इसे पूर्ण करने के लिए किया जाता है।
    • 51% answered correctly
    • Your first attempt
    • Your first attempt
    Re-attempt answer
    Click here to see the answer now
    Reattempt mode is Off. Turn it on from bottom bar

    Your First Attempt Answers

  • Accepted answer is between: 0 and 0
  • सही उत्तर स्विच है।

    Key Points

    • स्विच एक साधारण उपकरण है जिसका उपयोग या तो विद्युत परिपथ को वियोजित करने या इसे पूर्ण करने के लिए किया जाता है।
    • प्रतिरोधक एक निष्क्रिय विद्युत घटक है जो विद्युत धारा के प्रवाह का प्रतिरोध करता है।
    • ट्रांजिस्टर एक अर्धचालक उपकरण है जो प्रवर्धक और स्विच दोनों के रूप में कार्य करता है।
    • फ्यूज एक सुरक्षा विद्युत उपकरण है जो विद्युत धारा के बहुत अधिक होने पर इसे परिपथ से हटा देता है।

    Additional Information

    • ट्रांजिस्टर:
      • ट्रांजिस्टर अर्धचालक उपकरण होते हैं, जो आमतौर पर प्रवर्धक या विद्युत नियंत्रित स्विच के रूप में उपयोग किए जाते हैं। 
      • ट्रांजिस्टर आधारभूत निर्माण खंड हैं जो कंप्यूटर, मोबाइल फोन और अन्य सभी आधुनिक विद्युतीय परिपथ के संचालन को नियंत्रित करते हैं। 
      • सबसे महत्वपूर्ण ट्रांजिस्टर कार्य एक वर्तमान प्रवर्धक के रूप में और एक स्विच के रूप में होता है। 
      • ट्रांजिस्टर अक्सर प्रवर्धकों में विद्युत धारा प्रवर्धकों के रूप में  उपयोग किए जाते हैं जबकि ट्रांजिस्टर स्विच के रूप में अक्सर स्वचालित लाइट सर्किट में पाए जाते हैं। 
      • ट्रांजिस्टर के कार्य परिवर्तनीय प्रतिरोधक और स्विचिंग डिवाइस हैं।
    • संधारित्र:
      • संधारित्र एक ऐसा उपकरण है जो विद्युत ऊर्जा को विद्युत क्षेत्र में संग्रहित करता है।
      • यह दो टर्मिनलों वाला एक निष्क्रिय विद्युतीय घटक है।
      • संधारित्र के प्रभाव को धारिता के रूप में जाना जाता है।
      • धारिता: धारिता संधारित्र की आवेश संग्रहित करने की क्षमता है।
      • दो चालकों को एक विद्युतरोधी (परावैद्युत) द्वारा पृथक किया जाता है और जब एक विद्युत क्षेत्र आरोपित किया जाता है, तो उसमें विद्युत ऊर्जा एक आवेश के रूप में संग्रहित हो जाती है।
        • संधारित्र की धारिता (C): किसी चालक की धारिता उसके विभव (V) में वृद्धि से आवेश (Q) का अनुपात होता है, अर्थात
        • C = Q/V
    • प्रतिरोधक:
      • प्रतिरोधक एक निष्क्रिय तत्व होता है।
      • इसे 'R' द्वारा दर्शाया जाता है और इसे ओम (Ω) में मापा जाता है।
      • प्रतिरोधों का उपयोग धारा प्रवाह को कम करने, सिग्नल के स्तर को समायोजित करने, वोल्टेज को विभाजित करने के लिए किया जाता है।
    Question:
    बैक्टीरिया की खोज किसने की?
    • 72% answered correctly
    • Your first attempt
    • Your first attempt
    Re-attempt answer
    Click here to see the answer now
    Reattempt mode is Off. Turn it on from bottom bar

    Your First Attempt Answers

  • Accepted answer is between: 0 and 0
    • ​एंटोनी वान ल्यूवेनहॉक ने 1683 में मानव के मुंह से बैक्टीरिया की खोज की।
    • रॉबर्ट कोच ने टीबी बैक्टीरिया, माइकोबैक्टीरियम ट्यूबरकुलोसिस की खोज की और 1882 में अपनी जाँच को प्रकाशित किए।
    • अलेक्जेंडर फ्लेमिंग ने 1928 में पेनिसिलिन की खोज की।
    Question:
    इंटरनेशनल सिस्टम ऑफ यूनिट (SI) में एक हॉर्स पावर का विद्युत समतुल्य _________ के बराबर होता है
    • 88% answered correctly
    • Your first attempt
    • Your first attempt
    Re-attempt answer
    Click here to see the answer now
    Reattempt mode is Off. Turn it on from bottom bar

    Your First Attempt Answers

  • Accepted answer is between: 0 and 0
  • सही उत्तर 746 वाट है

    Key Points 

    • अश्वशक्ति: अश्वशक्ति एक इंजन की शक्ति की एक लोकप्रिय अत्युत्तम इकाई है। इसे किसी व्यक्ति या मशीन द्वारा प्रति इकाई समय पर किए गए कार्य के रूप में परिभाषित किया गया है। अश्वशक्ति के इस शब्द का आविष्कार जेम्स वाट ने किया था।
      • यह शक्ति की एक इकाई है जो एक मशीन की शक्ति की तुलना घोड़े की मांसपेशियों की शक्ति से करती है।
      • पहले के दिनों में इसका उपयोग भाप इंजन की शक्ति को मापने के लिए किया जाता था, इसकी तुलना खींचने वाले घोड़ों की शक्ति से की जाती थी। लेकिन बाद में, इसके उपयोग को विभिन्न प्रकार के पिस्टन इंजन, इलेक्ट्रिक मोटर्स, टर्बाइन और अन्य मशीनरी तक विस्तारित किया गया।
    • 550 फुट-पाउंड प्रति सेकंड की एक यांत्रिक अश्वशक्ति 745.7 वाट के बराबर है।

    ⇒ 1 Hp = 550 फुट पाउंड प्रति सेकंड

    ⇒ 1 पाउंड = 4044822 न्यूटन

    ⇒ 1 फुट = 0.3048 मीटर है

    ⇒ 1 Hp = 550 * 4.44822 * 0.3048 मीटर

    1Hp = 745.69 न्यूटन मीटर प्रति सेकंड= 746 वाट

    Question:

    यदि x2 - 7x + 12 = 0 है, तो x का मान ज्ञात कीजिये। 

    • 73% answered correctly
    • Your first attempt
    • Your first attempt
    Re-attempt answer
    Click here to see the answer now
    Reattempt mode is Off. Turn it on from bottom bar

    Your First Attempt Answers

  • Accepted answer is between: 0 and 0
  • दिया गया है:

    x2 - 7x + 12 = 0

    गणना:

    x2 - 7x + 12 = 0

    ⇒ x2 - 4x - 3x + 12 = 0

    ⇒ x(x - 4) - 3(x - 4)  = 0

    ⇒ (x - 4)(x - 3) = 0

    ∴ x = 4, 3

    ∴ x का मान 4, 3 है।

    Question:
    राजस्थान में रावतभाटा परमाणु ऊर्जा संयंत्र को किस देश के सहयोग से स्थापित किया गया है?
    • 12% answered correctly
    • Your first attempt
    • Your first attempt
    Re-attempt answer
    Click here to see the answer now
    Reattempt mode is Off. Turn it on from bottom bar

    Your First Attempt Answers

  • Accepted answer is between: 0 and 0
    • राजस्थान में स्थित रावतभाटा परमाणु ऊर्जा स्टेशन की स्थापना 1975 में हुई थी।
    • इसे कनाडा सरकार के सहयोग से स्थापित किया गया था।
    • यह भारत का दूसरा परमाणु ऊर्जा संयंत्र है।
    • भारत का पहला प्रमुख परमाणु ऊर्जा संयंत्र तारापुर महाराष्ट्र (1969 में स्थापित) में है।
    • रावतभाटा परमाणु ऊर्जा संयंत्र 1240 मेगावाट क्षमता वाला परमाणु संयंत्र है।
    • तारापुर परमाणु संयंत्र की स्थापना संयुक्त राज्य अमेरिका की सहायता से की गई थी।

    Question:
    प्लास्टर ऑफ पेरिस का रासायनिक सूत्र क्या है?
    • 77% answered correctly
    • Your first attempt
    • Your first attempt
    Re-attempt answer
    Click here to see the answer now
    Reattempt mode is Off. Turn it on from bottom bar

    Your First Attempt Answers

  • Accepted answer is between: 0 and 0
  • सही उत्तर (CaSO4).1/2H2है।

    • प्लास्टर ऑफ पेरिस को जिप्सम या कैल्शियम सल्फेट डाइहाइड्रेट को लगभग 140-180 डिग्री सेल्सियस पर गर्म करके प्राप्त किया जाता है। ऐसे तापमान पर गर्म होने पर जिप्सम प्लास्टर ऑफ पेरिस बनता है।
      • प्लास्टर ऑफ पेरिस का रासायनिक सूत्र  (CaSO4).1/2H2O है।
      • यह नाम पेरिस के मोंटमार्ट्रे पहाड़ी में जिप्सम की प्रचुर मात्रा होने के कारण लिया गया है।

    Additional Information

    • उपयोग:
      • प्लास्टर ऑफ़ पेरिस का उपयोग जातियों को डालने के लिए किया जाता है, प्रोस्थोडॉन्टिक रेस्टोरेशन के लिए मैट्रिस बनाया जाता है, कलाकारों को आर्टिकुलेटर्स से जोड़ा जाता है, और डेंटल प्रयोगशाला में सामान्य उपयोग के लिए जहाँ मजबूती महत्वपूर्ण नहीं है।
      • इसका उपयोग खिलौने, सजावटी सामग्री, सस्ते गहने, सौंदर्य प्रसाधन, ब्लैक-बोर्ड और मूर्तियों के लिए कास्ट बनाने में किया जाता है।
      • इसका उपयोग रसायन विज्ञान प्रयोगशालाओं में एयर गैप को सील करने के लिए किया जाता है, जहां वायु-रोधी व्यवस्था की आवश्यकता होती है।
      • इसका उपयोग पेंटिंग से पहले सतहों को चिकना बनाने के लिए किया जाता है।
    Question:
    यदि किसी अपरिवर्ती प्रतिरोधक के शिरों के बीच का विभांतर आधा कर दिया जाए, तो विद्युत शक्ति का मान होगा?
    • 18% answered correctly
    • Your first attempt
    • Your first attempt
    Re-attempt answer
    Click here to see the answer now
    Reattempt mode is Off. Turn it on from bottom bar

    Your First Attempt Answers

  • Accepted answer is between: 0 and 0
  • सही उत्तर एक चौथाई है। 

    Key Points

    • यदि किसी अपरिवर्ती प्रतिरोधक के सिरों के बीच विभवांतर को आधा कर दिया जाए, तो विद्युत शक्ति का मान एक चौथाई हो जाएगा।
      चूंकि हम जानते हैं कि,

    ⇒  Unknown node type: strongUnknown node type: strongUnknown node type: strongUnknown node type: strongUnknown node type: strong Unknown node type: strongUnknown node type: strongUnknown node type: strongUnknown node type: strongUnknown node type: strong

    • जहाँ, P विद्युत शक्ति है, V विभवांतर और R प्रतिरोधकता है। 
    • अतः जब,

    Vn = V/2 (आधा)

    ⇒ (Vn)2 = V2/4

    तब,

    ⇒ Pn = (Vn)2/R

    ⇒ Pn = V2/4R

    ⇒ Pn = V2/4R = P/4

    ⇒अतः शक्ति एक चौथाई हो जाती है।

     

    Question:
    भारत में मुद्रास्फीति को किससे मापा जाता है?
    • 8% answered correctly
    • Your first attempt
    • Your first attempt
    Re-attempt answer
    Click here to see the answer now
    Reattempt mode is Off. Turn it on from bottom bar

    Your First Attempt Answers

  • Accepted answer is between: 0 and 0
  • सही उत्तर उपभोक्‍ता मूल्‍य सूचकांक है

    Key Points 

    • भारत में मुद्रास्फीति के दो मुख्य संकेतक थोक मूल्य सूचकांक (WPI) और उपभोक्ता मूल्य सूचकांक (CPI) हैं।
    • कई विकासशील देश उपभोक्ता मूल्य सूचकांक (CPI) में परिवर्तन को मुद्रास्फीति के अपने केंद्रीय उपाय के रूप में उपयोग करते हैं।
    • भारत में, CPI (संयुक्त) को मुद्रास्फीति (अप्रैल 2014) को मापने के लिए नए मानक के रूप में घोषित किया गया है।
    • CPI संख्या को आम तौर पर मासिक रूप से मापा जाता है, और एक महत्वपूर्ण अंतराल के साथ, उन्हें नीतिगत उपयोग के लिए अनुपयुक्त बना देता है।
    • मुद्रास्फीति अर्थव्यवस्था में लंबी अवधि में वस्तुओं और सेवाओं की औसत मूल्यों में मापी गई वृद्धि होती है।
    • यह एक वृहद शब्द है जिसमें वस्तुओं के बड़े बाज़ार समूह पर मुद्रास्फीति का प्रभाव देखा जाता है।
    • जैसे-जैसे मुद्रा का मूल्य घटता है, मुद्रास्फ़ीति के संचयी प्रभाव की सूचना दी जाती है क्योंकि मुद्रा की क्रय शक्ति कम हो जाती है।

    Confusion Points

    • RBI ने 2014 से पहले अपने अधिकांश नीतिगत फैसलों के लिए WPI का इस्तेमाल किया था। WPI एक कमोडिटी बास्केट की संयुक्त मूल्य को दर्शाता है जिसमें 676 वस्तु शामिल हैं। लेकिन WPI में सेवाएँ शामिल नहीं हैं, और यह न तो उत्पादक और थोक व्यापारी के बीच और न ही थोक व्यापारी और खुदरा विक्रेता (उपभोक्ता) के बीच की बाधाओं को दर्शाता है।
    • इसलिए 2014 से, RBI गवर्नर रघु राम राजन द्वारा शुरू किए गए सुधारों के हिस्से के रूप में, RBI नीतिगत निर्णयों के लिए CPI में स्थानांतरित हो गया।

    Important Points 

    • मुद्रास्फीति:
      • अर्थव्यवस्था में लंबी अवधि के लिए वस्तुओं और सेवाओं की औसत कीमतों में गणना की गई वृद्धि को मुद्रास्फीति कहा जाता है।
      • यह एक वृहद अवधारणा है, जिसमें वस्तुओं के बड़े बाज़ार समूह पर मुद्रास्फीति का प्रभाव देखा जाता है।
      • मुद्रास्फीति का कुल प्रभाव दर्ज किया जाता है क्योंकि मुद्रा का मूल्य कम हो जाता है अर्थात मुद्रा की क्रय शक्ति कम हो जाती है।
      • रेंगती स्फीति, क्रियाशील स्फीति, तीव्र स्फीति और अति तीव्र स्फीति मुद्रास्फीति के प्रकार हैं।
      •  रेंगती स्फीति तब होती है जब मुद्रास्फीति की दर समय के साथ धीरे-धीरे बढ़ती है।
        • रेंगती स्फीति आम तौर पर 2% से 4% तक होती है।
      •  क्रियाशील स्फीति तब होती है जब मुद्रास्फीति एकल अंकों में होती है अर्थात 10% से कम।
        •  क्रियाशील स्फीति को केवल मध्यम मुद्रास्फीति के रूप में संदर्भित किया जा सकता है।
      • अति-तीव्र स्फीति को वर्धित मुद्रास्फीति के रूप में भी जाना जाता है।
        • तीव्र स्फीति मुद्रास्फीति को संदर्भित करती है जो असाधारण रूप से उच्च स्तर पर आगे बढ़ती है।"
        • तीव्र स्फीति 20% से 1000% के बीच की मुद्रास्फीति दर होती है। 
        • यह मुद्रास्फीति के चरम रूपों के लिए आरक्षित है - आमतौर पर 1,000% से अधिक।

    Additional Information 

    अंक

    थोक मूल्य सूचकांक

    उपभोक्ता मूल्य सूचकांक
    परिभाषा मात्रा थोक स्तर या बाजार मूल्य पर वस्तुओं की कीमतों में औसत परिवर्तन से संबंधित हैं। मात्राएं खुदरा स्तर या खुदरा बाजार मूल्य पर वस्तुओं की कीमतों में औसत परिवर्तन से संबंधित हैं।
    वस्तु  केवल वस्तुओं के लिए प्रतिबंधित वस्तु और सेवाओं दोनों के लिए प्रतिबंधित
    मुद्रास्फीति मापक  लेन-देन के प्रारंभिक चरण में, मुद्रास्फीति को मापा जाता है लेन-देन के अंतिम चरण में, मुद्रास्फीति को मापा जाता है
    आधार वर्ष 2011-12 2012
    Question:
    इनमें से कौन नालंदा विश्वविद्यालय के संस्थापक थे?
    • 80% answered correctly
    • Your first attempt
    • Your first attempt
    Re-attempt answer
    Click here to see the answer now
    Reattempt mode is Off. Turn it on from bottom bar

    Your First Attempt Answers

  • Accepted answer is between: 0 and 0
  • सही उत्‍तर कुमारगुप्त है।

    Key Points

    • नालंदा विश्वविद्यालय का निर्माण गुप्त शासक कुमारगुप्त ने करवाया था।
    • नालंदा विश्वविद्यालय बिहार में स्थित है।
    • यह बौद्ध शिक्षण का एक केंद्र था।
    • इसे अख्तियारुद्दीन बिन बख्तियार खिलजी ने बर्बाद कर दिया था।

    Additional Information

    धर्मपाल-:

    • विक्रमशिला विश्वविद्यालय की स्थापना धर्मपाल ने की थी।
    • यह बौद्ध शिक्षण का एक केंद्र था।
    • सोमपुरा महाविहार का निर्माण भी धर्मपाल ने करवाया था।

    गोपाल-:

    • ओदंतपुरी विश्वविद्यालय का निर्माण गोपाल द्वारा किया गया था।
    • यह मुख्य रूप से बौद्ध शिक्षण के प्रसार के उद्देश्य से स्थापित किया गया था।

    रामपाल-:

    • जगदला महाविहार की स्थापना रामपाल ने की थी।
    • यह बौद्ध शिक्षण का एक केंद्र था।
    Question:
    भारत के लिए कार्बन उत्सर्जन में शुद्ध-शून्य (net-zero) हासिल करने का लक्षित वर्ष क्या है?
    • 42% answered correctly
    • Your first attempt
    • Your first attempt
    Re-attempt answer
    Click here to see the answer now
    Reattempt mode is Off. Turn it on from bottom bar

    Your First Attempt Answers

  • Accepted answer is between: 0 and 0
  • सही उत्तर वर्ष 2070 है।

    In News

    • भारत ने वर्ष 2070 तक शुद्ध-शून्य (net-zero) हासिल करने का लक्ष्य रखा है।

    Key Points

    • अंतर्राष्ट्रीय नागरिक उड्डयन संगठन (ICAO) वैश्विक कार्यान्वयन सहायता संगोष्ठी 2023 में आयोजित एक उच्च-स्तरीय गोलमेज चर्चा में, भारत में नागरिक उड्डयन सचिव राजीव बंसल ने वर्ष 2070 तक कार्बन उत्सर्जन में शुद्ध शून्य (net-zero) बनने की देश की प्रतिबद्धता की घोषणा की।
    • यह लक्ष्य, हालांकि वर्ष 2050 के वैश्विक लक्ष्य से कम पड़ रहा था, लेकिन भारत द्वारा सामना की जा रही मौजूदा वास्तविकताओं और बुनियादी ढांचे की सीमाओं को देखते हुए इसे यथार्थवादी माना गया था।
    • वर्ष 2050 तक शुद्ध-शून्य (net-zero) प्राप्त करने के उड्डयन उद्योग के प्रयासों में निम्न कार्बन उत्सर्जन और SAF को अपनाने की दिशा में परिवर्तन को मौलिक कदम के रूप में पहचाना गया है।
    • वर्ष 2070 तक शुद्ध-शून्य (net-zero) लक्ष्य के प्रति भारत की प्रतिबद्धता विमानन में धारणीय प्रथाओं के महत्व की देश की मान्यता और वैश्विक पर्यावरणीय उद्देश्यों के साथ संरेखित करने के अपने दृढ़ संकल्प को प्रदर्शित करती है।

    Additional Information

    • अंतर्राष्ट्रीय नागरिक उड्डयन संगठन संयुक्त राष्ट्र की एक विशिष्ट एजेंसी है।
    • इसकी स्थापना 4 अप्रैल 1947 को हुई थी।
    • यह अंतरराष्ट्रीय हवाई मार्गदर्शन के सिद्धांतों और तकनीकों को बदलता है और सुरक्षित तथा व्यवस्थित विकास सुनिश्चित करने के लिए अंतरराष्ट्रीय हवाई परिवहन की योजना और विकास को बढ़ावा देता है।
    • मुख्यालय मॉन्ट्रियल, कनाडा में स्थित है।
    • ICAO के अध्यक्ष सल्वाटोर सिआचिटानो हैं।​
    Question:
    mRNA से संबद्ध राइबोसोम के समूह को क्या कहा जाता है?
    • 26% answered correctly
    • Your first attempt
    • Your first attempt
    Re-attempt answer
    Click here to see the answer now
    Reattempt mode is Off. Turn it on from bottom bar

    Your First Attempt Answers

  • Accepted answer is between: 0 and 0
  • सही उत्तर पॉलीसोम (बहुसूत्री) है।Key Points

    • पॉलीसोम (बहुसूत्री) के राइबोसोम 
      • राइबोसोम के रूप में जाने जाने वाले कोशिकीय अंग प्रोटीन और rRNA से बने होते हैं।
      • mRNA से बंधने से पहले, वे कोशिका द्रव्य में मुक्त रूप से उपस्थित होते हैं।
      • एकाधिक (बहु) राइबोसोम जो mRNA से संबद्ध होते हैं और जिनमें पॉलीपेप्टाइड शृंखलाएँ विकसित होती हैं, उन्हें पॉलीसोम (बहुसूत्री) के रूप में जाना जाता है।
      • एक पॉलीसोम कई राइबोसोम से बना होता है जो एक एकल mRNA को कई पॉलीपेप्टाइड्स (प्रोटीन) में एक बार उत्परिवर्तित करता है।
      • बहु राइबोसोम द्वारा पहुँचाया गया mRNA का कोडिंग भाग, एक बहुसूत्र का निर्माण करता है।
      • कोशिकाओं में इतने अधिक mRNA की उपस्थिति एक mRNA के कार्य में भाग लेने के लिए बहु राइबोसोम की क्षमता के कारण होती है।

    Additional Information

    • प्रोकैरियोटिक, यूकेरियोटिक, और झिल्ली-बद्ध बहुसूत्र सभी में बहु राइबोसोम की अलग-अलग संरचनाएं होती हैं जिन्हें बहुराइबोसोम (पॉलीराइबोसोम) भी कहा जाता है।
    • प्रोकैरियोटिक कोशिकाओं में जीवाणु बहुसूत्र दोहरी-पंक्ति संरचनाओं में पाए जाते हैं, और राइबोसोम छोटे उपइकाइयों के भीतर एक दूसरे के संपर्क में होते हैं।
    • तलीय दोहरी-पंक्ति बहुसूत्र और सघन व्यवस्थित 3D हेलिक्स, जो प्रोकैरियोटिक बहुसूत्र के समान हैं, यूकेरियोटिक कोशिकाओं में भी उपस्थित होती हैं।
    • पॉलीसोमल प्रोफाइलिंग पॉलीसोम गतिविधि की निगरानी के द्वारा जीन अभिव्यक्ति के स्तर को निर्धारित करने की एक विधि है।
    • पॉलीसोम (बहुसूत्री) प्रोफाइलिंग का उपयोग करके उत्परिवर्तन बंद कर दिया जाता है, और सुक्रोज ग्रेडिएंट का उपयोग परिणामी कोशिकाओं को अलग करने के लिए किया जाता है ताकि उन्हें अपकेंद्रीकरण (सेंट्रीफ्यूगेशन) द्वारा एकत्र किया जा सके।
    Question:
    निम्नलिखित में से कौन-सा अंगक केवल पादप में उपस्थित होता है?
    • 55% answered correctly
    • Your first attempt
    • Your first attempt
    Re-attempt answer
    Click here to see the answer now
    Reattempt mode is Off. Turn it on from bottom bar

    Your First Attempt Answers

  • Accepted answer is between: 0 and 0
  • सही उत्तर "लवक" है।

    Key Points

    • लवक सभी पादप की कोशिकाओं में पाए जाने वाले आणविक अंग हैं।
    • वे कुछ विशेष रंजकों से गुजरते हैं, जो पौधों को विशेष रंगों की आपूर्ति करते हैं।
    • लवक तीन प्रकार के होते हैं:
      • हरितलवक - इसमें पर्णहरित (हरा वर्णक) होता है)
      • वर्णिलवक - इसमें कैरोटीनॉयड (लाल, पीला और नारंगी वर्णक) होता है
      • ल्यूकोप्लास्ट - रंगहीन लवक होता है, मुख्य रूप से एक पादप की कोशिका में खाद्य पदार्थों को संग्रहीत करता है।

    Additional Information

    • लाइसोसोम
      • लाइसोसोम एक एकल झिल्ली का उपयोग करने और मजबूत पाचन एंजाइमों से युक्त छोटे थैली जैसी प्रणाली हैं।
      • लाइसोसोम को कोशिका की आत्महत्या की थैली के रूप में भी जाना जाता है।
    • माइटोकॉन्ड्रिया
      • माइटोकॉन्ड्रिया ट्यूब की तरह के अंग हैं जो एक दोहरी झिल्ली से घिरे होते हैं, जिसमें आंतरिक झिल्ली अत्यधिक मुड़ी हुई होती है।
      • माइटोकॉन्ड्रिया को नियमित रूप से कोशिका का "पावरहाउस" कहा जाता है।
    • रिक्तिकाएँ
      • रिक्तिकाएँ पादप, कवक और शैवाल कोशिकाओं के अंग हैं।
      • वे आंतरिक झिल्ली प्रणाली का हिस्सा हैं और एक टोनोप्लास्ट झिल्ली द्वारा बाकी कोशिका द्रव्य से अलग होते हैं।
      • कोशिका विस्तार में रिक्तिकाएँ महत्वपूर्ण भूमिका निभाती हैं, पोषक तत्वों के लिए भंडारण डिब्बों के रूप में कार्य करती हैं, और पाचन एंजाइमों वाले अपघट्य अंगों के रूप में कार्य करती हैं।
    Question:
    निम्नलिखित में से कौन सा 'नमस्ते (NAMASTE)' योजना का सही विस्तार है?
    • 16% answered correctly
    • Your first attempt
    • Your first attempt
    Re-attempt answer
    Click here to see the answer now
    Reattempt mode is Off. Turn it on from bottom bar

    Your First Attempt Answers

  • Accepted answer is between: 0 and 0
  • सही उत्तर विकल्प 1 है।

    Key Points

    • नमस्ते यंत्रीकृत स्वच्छता पारिस्थितिकी तंत्र के लिए राष्ट्रीय कार्रवाई (NAMASTE- नेशनल एक्शन फॉर मैकेनाइज्ड सेनिटेशन इकोसिस्टम) है।
    • इसे 2022 में आवास और शहरी मामलों के मंत्रालय और सामाजिक न्याय और अधिकारिता मंत्रालय द्वारा संयुक्त रूप से शुरू किया गया था।
    • इसका उद्देश्य असुरक्षित सीवर और सेप्टिक टैंक सफाई प्रथाओं को खत्म करना है।
    • केंद्रीय बजट 2023-24 ने नमस्ते योजना के लिए 100 करोड़ रुपये आवंटित किए हैं।

    Additional Information

    • नमस्ते योजना के उद्देश्य:
      • कुशल श्रमिकों द्वारा किए गए सभी स्वच्छता कार्य
      • स्वच्छता कार्य में शून्य मृत्यु
      • कोई भी सफाई कर्मचारी मानव मल के संपर्क में न आए।
      • स्वच्छता कार्यकर्ताओं की सुरक्षा में शहरी स्थानीय निकायों को मजबूत करना।
      • कुशल और पंजीकृत सफाई कर्मचारियों से सेवाएं लेने के लिए सफाई कर्मचारियों में जागरूकता बढ़ाना।
    Question:
    भारत की संविधान सभा का विचार पहली बार किसने दिया था?
    • 42% answered correctly
    • Your first attempt
    • Your first attempt
    Re-attempt answer
    Click here to see the answer now
    Reattempt mode is Off. Turn it on from bottom bar

    Your First Attempt Answers

  • Accepted answer is between: 0 and 0
  • सही उत्‍तर एम एन रॉय है

    • भारत की संविधान सभा का विचार पहली बार एम एन रॉय ने दिया था

    Key Points 

    • संविधान सभा का विचार 1934 में एम एन रॉय द्वारा प्रस्तावित किया गया था
      • कैबिनेट मिशन, 1946 संविधान के साथ-साथ एक अंतरिम सरकार गठित करने के लिए एक संविधान सभा की स्थापना की परिकल्पना की गई।
      • संविधान सभा को संविधान को अंतिम रूप देने में दो साल, ग्यारह महीने और अठारह दिन लगे।
      • सभा ने मसौदा या प्रारूप समिति के अध्यक्ष के रूप में डॉ. बी. आर. अम्बेडकर को 29 अगस्त, 1947 को नियुक्त किया

    Additional Information 

    • डॉ सच्चिदानंद सिन्हा संविधान सभा के पहले अध्यक्ष (अस्थायी) थे।
    • डॉ राजेंद्र प्रसाद संविधानसभा के स्थायी अध्यक्ष चुने गए
    • बी. एन. राव को संविधान सभा के संवैधानिक सलाहकार के रूप में नियुक्त किया गया था
    Question:
    10 सेमी त्रिज्या और 60° कोण वाले एक वृत्त के त्रिज्यखंड का क्षेत्रफल _________ है। [π = 3.14 लें]
    • 64% answered correctly
    • Your first attempt
    • Your first attempt
    Re-attempt answer
    Click here to see the answer now
    Reattempt mode is Off. Turn it on from bottom bar

    Your First Attempt Answers

  • Accepted answer is between: 0 and 0
  • दिया गया है:

    वृत्त की त्रिज्या = 10 सेमी

    कोण = 60º

    प्रयुक्त सूत्र:

    वृत्त के त्रिज्यखंड का क्षेत्रफल = Ø/360 πr2

    गणना:

    यहाँ,

    r = 10 सेमी

    Ø = 60º

    त्रिज्यखंड का क्षेत्रफल = 60/360 × 3.14 × 102 =  157/3 सेमी2

    ∴ वृत्त के त्रिज्यखंड का क्षेत्रफल 157/3 सेमी है। 

    Question:

    Direction: Fill in the blank with suitable preposition.

    We should abide _______ the rules of the organisation. 

    • 32% answered correctly
    • Your first attempt
    • Your first attempt
    Re-attempt answer
    Click here to see the answer now
    Reattempt mode is Off. Turn it on from bottom bar

    Your First Attempt Answers

  • Accepted answer is between: 0 and 0
  • The correct option is "by".

    Key Points

    • "By" is used when you want to identify the means or method of doing something.
      • For example: The snake was killed by a man.
    • Typically, "abide" is followed by the preposition "by" when it means to respect or comply with a rule, decision, or system.
      • For example: I don't want to abide by these ridiculous rules.
    • Similarly, in this context, 'by' is the appropriate preposition that should be used here.

    Therefore, the correct answer is: We should abide by the rules of the organisation.

    Additional Information

    • With: Grammatically, "with" is a preposition that is typically used to indicate association, company, relation, or possession.
      • Example: I have a meeting with my boss.
    • In: "In" is a preposition used to indicate situation or condition, location, inclusion within space, time or limits.
      • Example: I live in New York. (Location)
      • Example: I will submit the assignment in a week. (Time)
    • Of: "Of" is a preposition and it's typically used to express relationship, origin, belonging or possession, and it can also refer to a part or a quantity of something.
      • Example: The key of the house is under the mat. (Possession)
      • Example: A piece of cake. (Part of something)
    Question:
    एक समांतर श्रेढ़ी में पहले पद और अंतिम पद का योग 51 है। अंत से दूसरा पद 42 है। दूसरा पद ज्ञात कीजिए।
    • 47% answered correctly
    • Your first attempt
    • Your first attempt
    Re-attempt answer
    Click here to see the answer now
    Reattempt mode is Off. Turn it on from bottom bar

    Your First Attempt Answers

  • Accepted answer is between: 0 and 0
  • दिया गया है: पहले पद और अंतिम पद का योग 51 है। अंत से दूसरा पद 42 है।

    प्रयुक्त सूत्र:

    एक समांतर श्रेढ़ी का N वां पद, T= a + (n - 1)d, जहाँ ‘a’ पहला पद है, 'n' कुल पदों की संख्या है और 'd' सार्व अंतर है।

    गणना:

    माना पहला पद 'a' और सार्व अंतर 'd' है।

    प्रश्न से,

    a + Tn = 51

    ⇒ a + a + (n – 1)d = 51

    ⇒ 2a + (n – 1)d = 51     -----(1)

    अंत से दूसरा पद = 42

    T(n- 1 ) = 42

    ⇒ a + (n – 2)d = 42     ----(2)

    समीकरण (1) से (2) को घटाने पर हमें प्राप्त होता है,

    a + (n – 1)d – (n – 2)d = 9

    ⇒ a + nd –d –nd + 2d = 9

    ⇒ a + d = 9

    दूसरा पद = a + d = 9

    ∴ अभीष्ट उत्तर 9 है।

    Question:
    कोणार्क सूर्य मंदिर का निर्माण किसने कराया था?
    • 77% answered correctly
    • Your first attempt
    • Your first attempt
    Re-attempt answer
    Click here to see the answer now
    Reattempt mode is Off. Turn it on from bottom bar

    Your First Attempt Answers

  • Accepted answer is between: 0 and 0
    • कोणार्क सूर्य मंदिर पूर्वी गंगा राजवंश के राजा नरसिंह देव प्रथम द्वारा निर्मित 13वीं शताब्दी की संरचना है।
    • मौर्य साम्राज्य के राजा अशोक ने मध्य प्रदेश में सांची स्तूप का निर्माण किया था।
    • चोल राजवंश के राजाराजा चोल प्रथम ने तमिलनाडु के तंजावुर में बृहदेश्वर मंदिर का निर्माण किया।
    • विजय नगर साम्राज्य के राजा देवराय द्वितीय ने हम्पी, कर्नाटक में विठ्ठल मंदिर का निर्माण किया।

    Question:
    कौन सी नदी दो बार कर्क रेखा को पार करती है?
    • 68% answered correctly
    • Your first attempt
    • Your first attempt
    Re-attempt answer
    Click here to see the answer now
    Reattempt mode is Off. Turn it on from bottom bar

    Your First Attempt Answers

  • Accepted answer is between: 0 and 0
  • सही जवाब माही है।

    Key Points

    • माही नदी:
      • माही नदी मध्य प्रदेश में बहती है और राजस्थान और गुजरात राज्यों से बहने के बाद अरब सागर में मिल जाती है।
      • यह नदी है जो दो बार कर्क रेखा को पार करती है
    • नर्मदा नदी:
      • यह सबसे बड़ी पश्चिम बहने वाली नदी है।
      • यह उत्तर और दक्षिण भारत के बीच सीमा के रूप में कार्य करता है।
      • यह अमरकंटक पठार से निकलती है, जहाँ विंध्य और सतपुड़ा पर्वत श्रृंखलाएँ मिलती हैं।
      • यह अरब सागर में खाली होने से पहले मध्य प्रदेश, महाराष्ट्र और गुजरात राज्यों से होकर बहती है।
    • ताप्ती नदी:
      • यह मध्य भारत में एक नदी है, जो नर्मदा नदी के दक्षिण में स्थित है, और यह अरब सागर में जाती है।
      • ताप्ती नदी मध्य प्रदेश में स्थित है।
    • चंबल नदी:
      • यह मध्य प्रदेश में विंध्य पर्वतमाला में सिंगार चौरी चोटी पर स्थित है।
      • यह यमुना नदी की एक सहायक नदी है।
      • यह मध्य प्रदेश, उत्तर प्रदेश और राजस्थान राज्यों में स्थित है।
    Question:
    जब किसी वस्तु को लंबवत ऊपर की ओर फेंका जाता है, तो पिंड का संवेग क्या है?
    • 63% answered correctly
    • Your first attempt
    • Your first attempt
    Re-attempt answer
    Click here to see the answer now
    Reattempt mode is Off. Turn it on from bottom bar

    Your First Attempt Answers

  • Accepted answer is between: 0 and 0
  • सही उत्तर ऊंचाई में वृद्धि के साथ घटता है।

    Key Points

    • जब किसी वस्तु को ऊपर की ओर फेंका जाता है, तो वस्तु गुरुत्वाकर्षण के विरुद्ध जाती है और किसी बिंदु पर रुक जाती है।
      • अतः, वेग कम होता रहता है, जब तक कि यह 0 तक नहीं पहुंच जाता।
    • संवेग p = mv (जहाँ m = वस्तु का द्रव्यमान और v उसका वेग है)
      • अतः, संवेग p घटता रहता है और जब वेग 0 पर पहुँचता है तो 0 पर पहुँच जाता है।
    Question:
    अर्जुन पुरस्कार खेल और खेल में उत्कृष्ट उपलब्धि को मान्यता देने के लिए युवा मामले और खेल मंत्रालय द्वारा दिए जाते हैं। यह ________ में स्थापित किया गया था।
    • 56% answered correctly
    • Your first attempt
    • Your first attempt
    Re-attempt answer
    Click here to see the answer now
    Reattempt mode is Off. Turn it on from bottom bar

    Your First Attempt Answers

  • Accepted answer is between: 0 and 0
  • सही उत्‍तर 1961 है। 

    • भारत में अर्जुन पुरस्कार की शुरुआत 1961 में हुई थी।
    • अर्जुन पुरस्कार पहली बार 1961 में छह लोगों को प्रदान किया गया था।
    • मीना शाह (बैडमिंटन) 1962 में अर्जुन पुरस्कार पाने वाली पहली महिला थीं।
    • अर्जुन पुरस्कार के पहले प्राप्तकर्ता कृष्ण दास थे जिन्हें 1961 में तीरंदाजी के क्षेत्र में सम्मानित किया गया था।

    Additional Information

    • खेल और खेलों में उत्कृष्ट प्रदर्शन के लिए अर्जुन पुरस्कार 2021

    खिलाड़ी का नाम

    शिक्षण

    अर्पिन्दर सिंह 

    एथलेटिक्स

    सिंरंजित कौर 

    मुक्केबाज़ी 

    शिखर धवन 

    क्रिकेट 

    भवानी देवी चडलवादा आनंद सुंदररमन

    तलवारबाजी 

    मोनिका 

    हॉकी

    वंदना कटारिया 

    हॉकी

    संदीप नरवाल

    कबड्डी 

    हिमानी उत्तम पराब

    मलखंब

    अभिषेख शर्मा 

    निशानेबाजी 

    अंकिता रैना 

    टेनिस 

    दीपक पूनिया 

    कुश्ती 

    दिलप्रीत सिंह 

    हॉकी

    हरमन प्रीत सिंह 

    हॉकी

    रुपिंदर पाल सिंह 

    हॉकी

    सुरेन्द्र कुमार 

    हॉकी

    अमित रोहिदास 

    हॉकी

    बिरेन्द्र लाकरा 

    हॉकी

    सुमित 

    हॉकी

    नीलकांत शर्मा

    हॉकी

    हार्दिक सिंह 

    हॉकी

    विवेक सागर प्रसाद 

    हॉकी

    गुरजंत सिंह 

    हॉकी

    मंदीप सिंह 

    हॉकी

    शमशीर सिंह 

    हॉकी

    ललित कुमार उपाध्याय 

    हॉकी

    वरुण कुमार 

    हॉकी

    सिमरजीत सिंह 

    हॉकी

    योगेश कथुनिया

    पैरा एथलेटिक्स

    निषाद कुमार

    पैरा एथलेटिक्स

    प्रवीण कुमार

    पैरा एथलेटिक्स

    सुहाशयतिराज

    पैरा-बैडमिंटन

    सिंहराज अधान

    पैरा शूटिंग

    भविना पटेल 

    पैरा टेबल टेनिस

    हरविंदर सिंह 

    पैरा तीरंदाजी

    शरद कुमार 

    पैरा एथलेटिक्स

    Question:

    In the following question, out of the given four alternatives, select the alternative which best expresses the meaning of the Idiom/Phrase.

    To have the heart of stone 

    • 26% answered correctly
    • Your first attempt
    • Your first attempt
    Re-attempt answer
    Click here to see the answer now
    Reattempt mode is Off. Turn it on from bottom bar

    Your First Attempt Answers

  • Accepted answer is between: 0 and 0
  • The correct answer is 'Option 3' i.e. 'To feel no pity'.
    Key Points
    • The idiom "To have the heart of stone" (पत्थर का दिल होना) implies lacking compassion or being unfeeling.
    • Option 3, "To feel no pity" (दया नहीं आना), matches this meaning perfectly.
    Therefore, the correct answer is- 'To feel no pity'.
     Additional Information
    • Option 1, "To be fearless", and Option 4, "To have a hidden agenda", are not related to the idiom's meaning.
    • Option 2, "Not being able to make a decision", also doesn't relate to being unfeeling.
    Question:
    हाल ही में 'MISHTI' पहल खबरों में रही है। यह निम्नलिखित में से किससे संबंधित है?
    • 42% answered correctly
    • Your first attempt
    • Your first attempt
    Re-attempt answer
    Click here to see the answer now
    Reattempt mode is Off. Turn it on from bottom bar

    Your First Attempt Answers

  • Accepted answer is between: 0 and 0
  • सही उत्तर विकल्प 3 है।

    In News

    •  बजट 2023-24: वित्त मंत्री ने 'मिष्टी' पहल की घोषणा की।

    Key PointsMISHTI:

    • MISHTI, 'मैंग्रोव इनिशिएटिव फॉर शोरलाइन हैबिटैट्स एंड टैंजिबल इनकम' भारत द्वारा नवंबर 2022 में मिस्र में आयोजित जलवायु परिवर्तन पर संयुक्त राष्ट्र फ्रेमवर्क कन्वेंशन के 27वें सम्मेलन (COP27) के दौरान शुरू किए गए मैंग्रोव एलायंस फॉर क्लाइमेट में शामिल होने के बाद आया है।
    • वनीकरण में भारत की सफलता के आधार पर, वित्त मंत्री ने घोषणा की कि MGNREGS, CAMPA फंड और अन्य स्रोतों के बीच अभिसरण के माध्यम से, जहाँ भी संभव हो, समुद्र तट के किनारे और नमक की भूमि पर मैंग्रोव वृक्षारोपण के लिए 'MISHTIशुरू की जाएगी। अतः विकल्प 3 सही है।
    • 2021 वन सर्वेक्षण रिपोर्ट (FSR) के अनुसार, भारत में कुल मैंग्रोव कवर 4,992 वर्ग किलोमीटर है।
    • पिछली शताब्दी के दौरान देश ने अपने मैंग्रोव कवर का 40 प्रतिशत खो दिया।
    • उदाहरण के लिए, केरल ने पिछले तीन दशकों में अपने मैंग्रोव का 95 प्रतिशत खो दिया है।
    • इस गिरावट को कृषि, जलीय कृषि, पर्यटन और शहरी विकास के आवास के रूपांतरण के लिए जिम्मेदार ठहराया गया है।
    Question:
    अव्यवस्थित अक्षरों को उनके वास्तविक क्रम में व्यवस्थित कीजिए और उनमें से विषम ज्ञात कीजिए।
    • 28% answered correctly
    • Your first attempt
    • Your first attempt
    Re-attempt answer
    Click here to see the answer now
    Reattempt mode is Off. Turn it on from bottom bar

    Your First Attempt Answers

  • Accepted answer is between: 0 and 0
  • TOGA को GOAT के रूप में लिखा जाता है। 

    GEPONI को PIGEON के रूप में लिखा जाता है। 

    RARPOT को PARROT के रूप में लिखा जाता है। 

    LEEGA को EAGLE के रूप में लिखा जाता है। 

    GOAT को छोड़कर सभी उपरोक्त पक्षी हैं।

    अतः "TOGA" सही उत्तर है।

    Question:
    भुगतान शेष के चालू खाते में निम्नलिखित में से कौन शामिल नहीं है?
    • 26% answered correctly
    • Your first attempt
    • Your first attempt
    Re-attempt answer
    Click here to see the answer now
    Reattempt mode is Off. Turn it on from bottom bar

    Your First Attempt Answers

  • Accepted answer is between: 0 and 0
    • भुगतान संतुलन के चालू खाते में निवेश शामिल नहीं है।
    • भुगतान शेष का चालू खाता व्यापार संतुलन, शुद्ध कारक आय और शुद्ध स्थानान्तरण भुगतान का योग है।
    • BOP का चालू खाता या तो सकारात्मक अर्थ अधिशेष या नकारात्मक अर्थ आभाव है।
    • चालू खाते के चार घटक: व्यापार, शुद्ध आय, प्रत्यक्ष हस्तांतरण, संपत्ति आय हैं।
    Question:
    निम्नलिखित में से कौन-सा रोग मच्छर के काटने से नहीं होता है?
    • 70% answered correctly
    • Your first attempt
    • Your first attempt
    Re-attempt answer
    Click here to see the answer now
    Reattempt mode is Off. Turn it on from bottom bar

    Your First Attempt Answers

  • Accepted answer is between: 0 and 0
  • सही उत्तर विकल्प 4 है।

    Key Points

    • हेपेटाइटिस एक वायरल संक्रमण है, जो लीवर को प्रभावित करता है और इससे लीवर में सूजन और क्षति हो सकती है।
    • हेपेटाइटिस वायरस विभिन्न प्रकार के होते हैं, जैसे हेपेटाइटिस A, B, C, D और और प्रत्येक अलग-अलग तरीकों से फैलता है।
    • हेपेटाइटिस A और E आमतौर पर दूषित भोजन या जल के माध्यम से प्रेषित होते हैं, और वे दीर्घकालिक यकृत रोग का कारण नहीं बनते हैं।
    • हेपेटाइटिस B, C और D संक्रमित रक्त या शरीर के तरल पदार्थ के माध्यम से प्रेषित होते हैं, और वे दीर्घकालिक यकृत रोग का कारण बन सकते हैं और यकृत के खराब होने या यकृत कैंसर का कारण बन सकते हैं।
    • दूसरी ओर, मच्छरों को मलेरिया, डेंगू बुखार, ज़िका वायरस और पीले बुखार जैसी विभिन्न बीमारियों को प्रसारित करने के लिए जाना जाता है, लेकिन वे हेपेटाइटिस वायरस प्रसारित नहीं करते हैं।

    Additional Information

    • एलिफेंटियासिस एक परजीवी संक्रमण है, जो मच्छरों द्वारा फैलता है और लसीका प्रणाली को नुकसान के कारण शरीर के अंगों में पुरानी सूजन और वृद्धि का कारण बनता है।
      • इसे मच्छर नियंत्रण और संक्रमित व्यक्तियों के उचित उपचार जैसे उपायों से रोका जा सकता है।
    • डेंगू एक वायरल संक्रमण है, जो संक्रमित मच्छरों, मुख्य रूप से एडीज एजिप्टी मच्छर द्वारा मनुष्यों में फैलता है।
      • यह एक आम मच्छर जनित बीमारी है, जो विश्व के उष्णकटिबंधीय और उपोष्णकटिबंधीय क्षेत्रों में प्रचलित है।
    • मलेरिया प्लाज्मोडियम नामक परजीवी के कारण होने वाली एक गंभीर और कभी-कभी घातक बीमारी है, जो संक्रमित मादा एनोफिलीज मच्छरों के काटने से मनुष्यों में फैलती है।
      • मलेरिया विश्व के उष्णकटिबंधीय और उपोष्णकटिबंधीय क्षेत्रों में सबसे अधिक प्रचलित है, जिसमें अफ्रीका, एशिया और अमेरिका के कई देश शामिल हैं।
    Question:
    बौद्ध धर्म में "त्रिरत्न" का क्या अर्थ है?
    • 68% answered correctly
    • Your first attempt
    • Your first attempt
    Re-attempt answer
    Click here to see the answer now
    Reattempt mode is Off. Turn it on from bottom bar

    Your First Attempt Answers

  • Accepted answer is between: 0 and 0
  • सही उत्तर है बुद्ध, धम्म (धर्म), संघ

    Key Pointsसंस्कृत में त्रिरत्न का अर्थ है 'तीन रत्न'

    • बुद्ध
    • धम्म (धर्म): उनकी शिक्षा
    • संघ: उन सभी का समुदाय जो शिक्षाओं का पालन करते हैं।

    बुद्ध धर्म

    • सिद्धार्थ गौतम ("बुद्ध") द्वारा सिद्दांत स्थापित किया गया था।
    • सिद्धार्थ गौतम, भगवान बुद्ध का जन्म 563 ईसा पूर्व में नेपाल के लुम्बिनी में हुआ था
    • बोध गया में एक पीपल के पेड़ के नीचे निर्वाण प्राप्त किया और इसलिए बुद्ध (एक प्रबुद्ध) के रूप में जाने जाते थे।
    • सारनाथ (बनारस) में अपना पहला उपदेश दिया, जिसे धर्म चक्र प्रवर्तन कहा जाता है।
    • बुद्ध का 80 वर्ष की आयु में कुशीनगर (U.P) में निधन हो गया

    बुद्ध के महान सत्य

    • संसार दुःख से भरा है।
    • लोग इच्छाओं के कारण पीड़ित होते हैं
    • यदि इच्छाओं पर विजय प्राप्त की जाती है निर्वाण प्राप्त किया जा सकता है अर्थात्, जन्म और मृत्यु के चक्र से मुक्त होने के लिए  8 पथ (अष्टांगिका मार्ग) का पालन किया जा सकता है
      • सम्यक दृष्टि
      • सम्यक संकल्प
      • सम्यक वाक 
      • सम्यक कर्म 
      • सम्यक जीविका
      • सम्यक व्यायाम
      • सम्यक स्मृति
      • सम्यक समाधि

    बुद्ध के उपदेश

    • बुद्ध एक व्यावहारिक सुधारक थे और आत्मा या ईश्वर या आध्यात्मिक दुनिया में विश्वास नहीं करते थे और खुद को दुनिया की समस्याओं से संबंधित उपाय के उपदेश देते थे।
    • उनका उपदेश था कि एक व्यक्ति को विलासिता, और मितव्ययिता, और एक मध्य मार्ग निर्धारित दोनों की अधिकता से बचना चाहिए।
    • उन्होंने कर्म (वर्ण जन्म पर नहीं कर्म पर आधारित है ) और अहिंसा पर बड़ा जोर दिया।
    • वर्ण व्यवस्था का विरोध किया और सामाजिक समानता के सिद्धांत को रखा।
    • बौद्ध ग्रन्थ 
      • त्रिपिटक: सभी पाली भाषा में लिखे गए 
      • सुत्त-पिटक
      • विनय-पिटक
      • अभिधम्म-पिटक
    • बौद्ध परिषद
    परिषद् स्थान

    काल

    अध्यक्षता राजा परिणाम
    पहली परिषद् राजगीर, सप्तपर्णी गुफा में 483 ई.पू. बुद्ध की मृत्यु के तुरंत बाद  महाकश्यप अजातशत्रु आनंद की रचना: सुत्तपिटक (बुद्ध की शिक्षा) और उपाली ने विनयपिटिका (बौद्ध धर्म के मठ कोड) की रचना की
    दूसरी परिषद् वैशाली 383 ई.पू. बुद्ध की मृत्यु के बाद लगभग 100 ईसा पूर्व  सबकामी कालाशोक इस परिषद ने विनय पिटक और अनुशासन संहिता पर विवादों का निपटारा किया।
    तीसरी परिषद् पाटलिपुत्र 250 ई.पू.

    मोगलीपुत्त तिस्स

    अशोक अभिधम्म पिटक का संकलन (बौद्ध धर्म का दार्शनिक विस्तार) हुआ
    चौथी परिषद् कश्मीर, कुंडलवन में  72 ई वसुमित्र कनिष्क हीनयान और महायान में बौद्ध धर्म के विभाजन के परिणामस्वरूप
    Question:
    निम्नलिखित में से कौन सा प्राकृतिक तंतु नहीं है?
    • 51% answered correctly
    • Your first attempt
    • Your first attempt
    Re-attempt answer
    Click here to see the answer now
    Reattempt mode is Off. Turn it on from bottom bar

    Your First Attempt Answers

  • Accepted answer is between: 0 and 0
  • सही उत्तर रेयॉन है।

    Key Points

    • प्राकृतिक तंतु पौधों, कीड़ों और कृमियों से प्राप्त होते हैं।
    • कपास, ऊन, लिनन और रेशम प्राकृतिक तंतुओं के उदाहरण हैं।
      • अतः रेयॉन प्राकृतिक तंतु नहीं है।
    • सभी तंतुओं को खींचकर लंबा किया जाता और काता जाता है, जिससे लंबे धागे बनते हैं जिन्हें सूत कहा जाता है
    • इसे कताई कहा जाता है।

    Important Points 

    • सूत को कपड़े के उत्पादन के लिए करघे नामक मशीन पर बुना जाता है। 
      • कपास के तंतु कपास के पौधे के फल से प्राप्त होते हैं, जिसे कपास गोलक कहा जाता है।
      • ऊनी तंतु भेड़, खरगोश और बकरियों की ऊन से प्राप्त होते हैं।
      • रेशम के तंतु रेशम के कीड़ों के कोकून से प्राप्त होते हैं।

    Additional Information

    • कृत्रिम रेशे​
      • ऐसे रेशे जो मानव निर्मित होते हैं, कृत्रिम रेशे (संश्लिष्ट रेशे) कहलाते हैं। कृत्रिम रेशे कारखानों में बनते हैं।
      • रेयॉन, टेरीक्लॉथ, टेरिलीन और नायलॉन कुछ लोकप्रिय कृत्रिम रेशे हैं। 
    Question:
    लिटमस विलयन एक बैंगनी डाई है, जो लाइकेन से निकाला जाती है, जो कि ________ खंड से संबंधित एक पौधा है।
    • 31% answered correctly
    • Your first attempt
    • Your first attempt
    Re-attempt answer
    Click here to see the answer now
    Reattempt mode is Off. Turn it on from bottom bar

    Your First Attempt Answers

  • Accepted answer is between: 0 and 0
  • सही उत्‍तर थैलोफाइटा है

    • लिटमस घोल एक बैंगनी डाई है, जिसे लाइकेन से निकाला जाता है, जो थैलोफाइटा खंड से संबंधित एक पौधा है, और आमतौर पर एक संकेतक के रूप में उपयोग किया जाता है।
      • जब लिटमस का घोल न तो अम्लीय होता है और न ही क्षारीय, इसका रंग बैंगनी होता है।
      • लिटमस का मुख्य उपयोग यह परीक्षण करना है कि कोई घोल अम्लीय है या क्षारीय, क्योंकि नीला लिटमस पेपर अम्लीय परिस्थितियों में लाल हो जाता है, और लाल लिटमस पेपर क्षारीय या क्षारीय परिस्थितियों में नीला हो जाता है।
      • न्यूट्रल लिटमस पेपर बैंगनी होता है।

    Additional Information

    • पादप जगत का वर्गीकरण
      • पादप जगत को पाँच उपसमूहों में वर्गीकृत किया गया है:
        • थैलोफाइटा: थैलोफाइटा में आदिम और सरल शरीर संरचना वाले पौधे शामिल हैं।
          • पौधे का शरीर एक थैलस है, वे फिलामेंटस, औपनिवेशिक, शाखित या अशाखित हो सकते हैं।
          • उदाहरणों में हरी शैवाल, लाल शैवाल और भूरे शैवाल शामिल हैं।
          • सामान्य उदाहरण वॉल्वॉक्स, फुकस, स्पाइरोगाइरा, चरा, पॉलीसिफोनिया, उलोथ्रिक्स आदि हैं।
        • ब्रायोफाइटा: ब्रायोफाइट्स में संवहनी ऊतक नहीं होते हैं।
          • पौधे के शरीर में जड़ जैसी, तना जैसी और पत्ती जैसी संरचनाएँ होती हैं।
          • ब्रायोफाइट्स स्थलीय पौधे हैं लेकिन उन्हें "पादप जगत के उभयचर" के रूप में जाना जाता है क्योंकि उन्हें यौन प्रजनन के लिए पानी की आवश्यकता होती है।
          • वे नम और छायादार स्थानों में मौजूद हैं। ब्रायोफाइटा में काई, हॉर्नवॉर्ट्स और लिवरवॉर्ट्स शामिल हैं।
          • कुछ सामान्य उदाहरण हैं मर्चेंटिया, फनरिया, स्फाग्नम, एंथोसेरोस आदि।
        • टेरिडोफाइटा: टेरिडोफाइट्स में जड़, तना और पत्तियों में एक अच्छी तरह से विभेदित पौधे का शरीर होता है।
          • उनके पास पानी और अन्य पदार्थों के संचालन के लिए एक संवहनी प्रणाली है।
          • कुछ सामान्य उदाहरण सेलाजिनेला, इक्विसेटम, पर्टिस आदि हैं।
        • जिम्नोस्पर्म: जिम्नोस्पर्म में एक अच्छी तरह से विभेदित पौधे का शरीर और संवहनी ऊतक होता है।
          • वे नग्न बीज धारण करते हैं, अर्थात बीज एक फल के भीतर संलग्न नहीं होते हैं।
          • जिम्नोस्पर्म के कुछ सामान्य उदाहरण साइकस, पिनस, एफेड्रा आदि हैं।
        • एंजियोस्पर्म: एंजियोस्पर्म एक अच्छी तरह से विभेदित पौधे के शरीर वाले बीज-असर वाले संवहनी पौधे हैं।
          • एंजियोस्पर्म के बीज फलों के भीतर संलग्न होते हैं।
    Question:
    इन प्रश्नों में एक वाक्य को तीन भागों में विभक्त किया गया है। जिसमें से एक भाग में त्रुटि हो सकती है। यदि है तो उस भाग का चयन करें, अन्यथा (D) का चयन करें। 
    • 73% answered correctly
    • Your first attempt
    • Your first attempt
    Re-attempt answer
    Click here to see the answer now
    Reattempt mode is Off. Turn it on from bottom bar

    Your First Attempt Answers

  • Accepted answer is between: 0 and 0
  • दिए गए प्रश्न में "सड़क उपर' भाग में  त्रुटि है। यहाँ पर 'सड़क उपर' की जगह 'सड़क के ऊपर' सही होगा। 

    • सही वाक्य होगा-
      • घोड़ा गाड़ी सड़क के ऊपर दौड़ रही थी। 

    Key Points

    अशुद्ध वाक्य

    शुद्ध वाक्य

    वह लगभग दौड़ रहा है। 

    वह दौड़ रहा है।

    हम सब परस्पर आपस में बराबर हैं। 

    हम सब आपस में बराबर हैं।

    उसके बाद फिर क्या हुआ

    उसके बाद क्या हुआ।

     

    Additional Information

     
    वाक्‍य में अशुद्धियाँ

    • वाक्‍य सम्‍प्रेषण की सबसे महत्‍वपूर्ण और सार्थक इकाई है अत: वाक्‍यगत अशुद्धियों को शुद्ध रूप में लिखना वाक्‍य को अधिक सरल बनाता है। 
    • वाक्‍य में पदक्रम, पुनरावृत्ति, वर्तनी, शब्‍द-अर्थ, संज्ञा, सर्वनाम, लिंग, वचन, क्रिया, विशेषण, क्रिया-विशेषण आदि अशुद्धियॉं हो सकती हैं। 
    Question:
    प्रकाश संश्लेषण निम्नलिखित कारकों से प्रभावित होता है:
    • 41% answered correctly
    • Your first attempt
    • Your first attempt
    Re-attempt answer
    Click here to see the answer now
    Reattempt mode is Off. Turn it on from bottom bar

    Your First Attempt Answers

  • Accepted answer is between: 0 and 0
  • Key Points

    • विकल्प में बताए गए सभी कारक जैसे जल, ऑक्सीजन, कार्बन डाइऑक्साइड, तापमान प्रभावी प्रकाश संश्लेषण के लिए महत्वपूर्ण हैं। 
    • लेकिन नाइट्रोजन ऐसा कारक नहीं है जो प्रकाश संश्लेषण को  सीधे प्रभावित करता है।

    अत:, सही विकल्प (1) CO2 + तापमान + प्रकाश हैImportant Points

    • प्रकाश संश्लेषण वह प्रक्रिया होती है जिसके द्वारा पौधे शर्करा के रूप में ऑक्सीजन और ऊर्जा बनाने के लिए सूर्य के प्रकाश, जल और कार्बन डाइऑक्साइड का उपयोग करते हैं।
    • प्रकाश संश्लेषण को प्रभावित करने वाले निम्नलिखित कारक हैं:
    1. कार्बन डाइऑक्साइड
      • यदि प्रकाश की तीव्रता और तापमान अनुकूल होते हैं, तो CO2 प्रकाश संश्लेषण की दर को सीमित कर देता है।
      • CO2 के निचले स्तर पर, CO2 की सांद्रता में वृद्धि से प्रकाश संश्लेषण की दर बढ़ जाती है।
      • उच्च सांद्रता में दर में गिरावट देखी गई है। 
      • वातावरण में CO2 की मात्रा 0.03% होती है।  
      • यदि CO2 की मात्रा 0.1% तक बढ़ जाती है, तो प्रकाश संश्लेषण की दर बढ़ जाती है। 
      • यदि CO2 की मात्रा 0.1% से अधिक बढ़ जाती है, तो प्रकाश संश्लेषण की दर घट जाती है। 
      • कार्बन डाइऑक्साइड का उच्च स्तर पौधों के लिए हानिकारक होता है। 
      • CO2 पौधे में एकत्र होने लगती है और प्रकाश संश्लेषण को रोकती है। 
    2. प्रकाश 
      • प्रकाश की तीव्रता​
        • पूर्ण सूर्य के प्रकाश तक प्रकाश की तीव्रता के साथ प्रकाश संश्लेषण की दर लगातार बढ़ती जाती है
        • आंतरिक पत्तियों की आंशिक रोशनी के कारण अधिकतम प्रकाश संश्लेषण के लिए उच्च प्रकाश तीव्रता की आवश्यकता होती है। 
        • उच्च तीव्रता पर पौधे का तापमान बढ़ जाता है।
        • इससे वाष्पोत्सर्जन बढ़ जाता है, रंध्र बंद हो जाते हैं और CO2 की मात्रा कम हो जाती है। 
      • प्रकाश गुणवत्ता
        • 400 nm और 700 nm की तरंग दैर्ध्य के बीच का प्रकाश सबसे प्रभावी होता है, जिसे प्रकाश संश्लेषक रूप से सक्रिय विकिरण (PAR) कहा जाता है।
        • लाल और नीला प्रकाश प्रकाश संश्लेषण में प्रभावी होता हैं। 
      तापमान 
      • प्रकाश संश्लेषण उच्च तापमान के प्रति संवेदनशील होता है क्योंकि उच्च तापमान एंजाइम की निष्क्रियता का कारण बनता है। 
      • प्रकाश संश्लेषण की दर एक बिंदु (सामान्यतः 25°C) तक तापमान में वृद्धि के साथ बढ़ जाती है जो पौधों के प्रकार में भिन्न होती है। 
      • इस सीमा के ऊपर, जीवद्रव्य पर उच्च तापमान के हानिकारक प्रभावों से प्रकाश संश्लेषण की दर में तीव्रता से कमी आती है। 
    3. जल 
      • जड़ों द्वारा अवशोषित जल का लगभग 1% या उससे कम ही वास्तव में प्रकाश संश्लेषण में उपयोग होता है
      • जल की कमी वाली मृदा में प्रकाश संश्लेषण की दर में कमी होती है
      • पौधे में जल की कमी से रंध्र बंद हो जाते हैं और CO2 का अवशोषण कम हो जाता है। 
    4. ऑक्सीजन सघनता -
      • ऑक्सीजन की सघनता में वृद्धि से प्रकाश संश्लेषण की दर में कमी आती है। 
      • ऑक्सीजन प्रकाश संश्लेषण में बनने वाले मध्यस्थ को तोड़ देता है, CO2 के साथ मिलकर रूबिस्को के साथ प्रतिस्पर्धा करता है। 

    Additional Information

    ब्लैकमैन का सीमित कारक का सिद्धांत -

    यह सिद्धांत बताता है कि जब कोई प्रक्रिया एक से अधिक कारकों द्वारा नियंत्रित होती है, तो प्रक्रिया की दर उस कारक द्वारा नियंत्रित होती है जो उसके न्यूनतम मूल्य के सबसे नजदीक होती है।

    • प्रक्रिया - जैसे प्रकाश संश्लेषण
    • एक से अधिक कारक - जल, ऑक्सीजन, कार्बन डाइऑक्साइड, तापमान

    प्रकाश संश्लेषण

    • यदि कोई पौधा स्थिर तापमान के साथ एक निश्चित मात्रा में प्रकाश की तीव्रता के संपर्क में आता है लेकिन CO2 कम उपलब्ध होती है। 
    • प्रकाश की तीव्रता या उच्चतम इष्टतम तापमान में वृद्धि के साथ प्रकाश संश्लेषण की दर में वृद्धि नहीं होगी। 
    • अतः, इस स्थिति में, CO2 सीमित कारक होती है। 
    Question:
    प्रकाश के तरंग सिद्धांत का जनक किसे माना जाता है?
    • 51% answered correctly
    • Your first attempt
    • Your first attempt
    Re-attempt answer
    Click here to see the answer now
    Reattempt mode is Off. Turn it on from bottom bar

    Your First Attempt Answers

  • Accepted answer is between: 0 and 0
  • सही उत्तर क्रिश्चियन हाइगेन्स है

    Key Points

    • प्रकाश का तरंग सिद्धांत क्रिश्चियन हाइगेन्स द्वारा प्रस्तावित किया गया था। 
    • 1678 में, हाइगेन्स ने प्रस्तावित किया कि हर बिंदु जो एक चमकदार अवरोध से टकराती है, गोलाकार तरंग के स्रोत में बदल जाती है। 
    • द्वितीयक तरंगों का योग, जो विक्षोभ का परिणाम है, यह निर्धारित करता है कि नई तरंग किस रूप में होगी। 
    • प्रकाश के इस सिद्धांत को 'हाइगेन्स' सिद्धांत' के रूप में जाना जाता है।
    Additional Information

    अल्बर्ट आइंस्टीन:

    • आइंस्टीन ने अपने विशेष सापेक्षता के सिद्धांत से दिखाया कि द्रव्यमान को ऊर्जा के दूसरे रूप के रूप में मानना ​​​​आवश्यक है।
    • विशेष सापेक्षता के इस सिद्धांत के आगमन से पहले, यह माना जाता था कि अभिक्रिया में द्रव्यमान और ऊर्जा अलग-अलग संरक्षित होते हैं।
    • आइंस्टीन ने निम्न प्रसिद्ध द्रव्यमान-ऊर्जा तुल्यता संबंध दिया,
      • E = mc2

    गैलीलियो गैलीली:

    • गैलीलियो एक प्राकृतिक दार्शनिक, खगोलशास्त्री और गणितज्ञ थे।
    • उन्होंने क्रांतिकारी दूरबीन की खोज भी की, जिसमें बृहस्पति के चार सबसे बड़े उपग्रह भी शामिल हैं। 

    जकारियास जेनसन:

    • यौगिक सूक्ष्मदर्शी का आविष्कार करने का श्रेय जकारियास जैनसेन को दिया जाता है।
    Question:
    सर्वोच्च न्यायालय में तदर्थ न्यायाधीशों की नियुक्ति कौन कर सकता है?
    • 14% answered correctly
    • Your first attempt
    • Your first attempt
    Re-attempt answer
    Click here to see the answer now
    Reattempt mode is Off. Turn it on from bottom bar

    Your First Attempt Answers

  • Accepted answer is between: 0 and 0
  • इसका उत्तर है राष्ट्रपति की पूर्व सहमति से भारत के मुख्य न्यायाधीश

    • भारत के संविधान का अनुच्छेद 127 सर्वोच्च न्यायालय में तदर्थ न्यायाधीशों की नियुक्ति का प्रावधान करता है।
      • इसमें कहा गया है कि भारत के मुख्य न्यायाधीश, राष्ट्रपति की पूर्व सहमति से, किसी उच्च न्यायालय के न्यायाधीश को सर्वोच्च न्यायालय के न्यायाधीश के रूप में कार्य करने के लिए नियुक्त कर सकते हैं, यदि उस समय सर्वोच्च न्यायालय में न्यायाधीशों की संख्या उपलब्ध हो उच्चतम न्यायालय के किसी भी सत्र को आयोजित करने या जारी रखने के उद्देश्य से निर्धारित कोरम से कम है।
    • भारत के मुख्य न्यायाधीश को उस उच्च न्यायालय से तदर्थ न्यायाधीश नियुक्त करने से पहले संबंधित उच्च न्यायालय के मुख्य न्यायाधीश से परामर्श करना पड़ता है।
    • तदर्थ न्यायाधीश के पास सर्वोच्च न्यायालय के स्थायी न्यायाधीश के समान शक्तियाँ और विशेषाधिकार होंगे।
    • भारत के राष्ट्रपति को एक तदर्थ न्यायाधीश की नियुक्ति के लिए अपनी सहमति देनी होती है, लेकिन वह भारत के मुख्य न्यायाधीश के निर्णय को वीटो नहीं कर सकते।
    Question:
    निर्देश : निम्नलिखित प्रत्येक चार विकल्पों में से बेमेल (असंगत) शब्द चिह्नित कीजिए:
    • 64% answered correctly
    • Your first attempt
    • Your first attempt
    Re-attempt answer
    Click here to see the answer now
    Reattempt mode is Off. Turn it on from bottom bar

    Your First Attempt Answers

  • Accepted answer is between: 0 and 0
  • सही उत्तर है - ‘आधि’।
    • आधि’ शब्द बेमेल (असंगत) शब्द है।

    Key Points

    • अन्य विकल्पों शारीरिक,दैहिक, और मानसिक में से ‘इक’ प्रत्यय का प्रयोग हुआ है।
    • अत: ‘आधि’ सही विकल्प होगा।

    Additional Informationप्रत्यय : 

    • वे शब्दांश जो किसी शब्द या धातु के अंत में जुड़कर उसके अर्थ को प्रभावित कर देते हैं उन्हें प्रत्यय कहते हैं।

    जैसे :

    • कृ + तव्य = कर्तव्य
      यहाँ ‘तव्य’ प्रत्यय का प्रयोग हुआ है।
    • महात्मा + = महात्म्य
      यहाँ ‘’ प्रत्यय का प्रयोग हुआ है।

    उपसर्ग :

    • शब्दांश या अव्यय जो किसी शब्द के पहले आकर उसका विशेष अर्थ बनाते हैं, उपसर्ग कहलाते हैं।

    जैसे :

    • प्र + चार = प्रचार
      इसमें ‘प्र’ उपसर्ग है, जो चार शब्द के पहले जुड़ा है।
    • अनु + शासन = अनुशासन
      इसमें ‘अनु’ उपसर्ग है, जो शासन शब्द के पहले जुड़ा है।
    Question:
    जनता का ___________ लाभ हर पार्टी उठाती है। रिक्त स्थान हेतु उपयुक्त शब्द होगा?
    • 60% answered correctly
    • Your first attempt
    • Your first attempt
    Re-attempt answer
    Click here to see the answer now
    Reattempt mode is Off. Turn it on from bottom bar

    Your First Attempt Answers

  • Accepted answer is between: 0 and 0
  • रिक्त स्थान के लिए उपयुक्त शब्द ‘राजनैतिक´ होगा। अतः इसका सही उत्तर विकल्प 4 राजनैतिकहोगा। अन्य विकल्प सही उत्तर नहीं हैं।

    स्पष्टीकरण:

    ‘राजनैतिक’ लाभ इस वाक्य के लिए उपयुक्त शब्द है क्योंकि मुफ्त, अर्थव्यवस्था और राजनीतिकरण शब्द वाक्यानुरूप नहीं हैं। इसलिए ये सभी अनुचित हैं।
    Question:

    एक समबाहु त्रिभुज की भुजा 4 सेमी है। इसका क्षेत्रफल क्या है?

     

    • 71% answered correctly
    • Your first attempt
    • Your first attempt
    Re-attempt answer
    Click here to see the answer now
    Reattempt mode is Off. Turn it on from bottom bar

    Your First Attempt Answers

  • Accepted answer is between: 0 and 0
  • समबाहु त्रिभुज का क्षेत्रफल  =34(side)2

    ⇒समबाहु त्रिभुज का क्षेत्रफल  =34(4)2

    ⇒समबाहु त्रिभुज का क्षेत्रफल =43cm2

    ∴ समबाहु त्रिभुज का क्षेत्रफल 4√3 सेमीहै ।
    Question:

    Choose the word that is opposite in meaning to the given word.

    Proceed

    • 47% answered correctly
    • Your first attempt
    • Your first attempt
    Re-attempt answer
    Click here to see the answer now
    Reattempt mode is Off. Turn it on from bottom bar

    Your First Attempt Answers

  • Accepted answer is between: 0 and 0
  • The correct answer is 'Option 3' i.e. 'Withdraw'.
    Key Points
    • The word "Proceed" means to begin or continue a course of action.(आगे बढ़ना)
    • The word that is opposite in meaning to "Proceed" is: 'Withdraw'.
    • "Withdraw" means to stop participating in an activity or to remove from a place. (वापस लेना)
    Therefore, the correct answer is: 'Withdraw'.
    Additional Information
    • Accidental: This refers to something happening by chance, unexpectedly, or unintentionally. (आकस्मिक)
    • Local: This refers to something that belongs to a particular area or neighborhood.(स्थानीय)
    • Wide: This term refers to something having a great extent from side to side. (चौड़ा)
    Question:
    वह तापमान जिसपर एक ठोस वायुमंडलीय दबाव पर द्रव्य बनने के लिए पिघलता है, इसका गलनांक कहलाता है। तो ठोस का गलनांक क्या दर्शाता है?
    • 33% answered correctly
    • Your first attempt
    • Your first attempt
    Re-attempt answer
    Click here to see the answer now
    Reattempt mode is Off. Turn it on from bottom bar

    Your First Attempt Answers

  • Accepted answer is between: 0 and 0
  • संकल्पना -

    विगलन और गलनांक - 

    • ठोस से द्रव्य तक अवस्था के परिवर्तन की प्रक्रिया विगलन कहलाता है।
    • वह तापमान जिसपर द्रव्य पिघलना शुरू करता है, उसे ठोस के गलनांक के रूप में जाना जाता है।
    • वायुमंडलीय दबाव पर एक पदार्थ के गलनांक को सामान्य गलनांक कहा जाता है।

    वर्णन -

    • ठोस के गलनांक को उस तापमान के रूप में परिभाषित किया जाता है जिसपर ठोस एक वायुमंडल के बाहरी दबाव के तहत इसके द्रव्य के साथ समतुल्यता में मौजूद होता है।
    • ठोस में अणुओं के बीच आकर्षण का बल बहुत अधिक होता है और अणु ठोस के अंदर उनकी स्थिति में लगभग निर्दिष्ट होते हैं।
    • जैसे-जैसे तापमान बढ़ता है, ठोस अवस्था द्रव्य अवस्था में परिवर्तित होना शुरू हो जाता है। इसलिए, अणुओं के बीच आकर्षण का बल कमजोर बन जाता है और अणु द्रव्य के आयतन के अंदर मुक्त रूप से गति कर सकते हैं। हालाँकि, वे सतह के बाहर आने में सक्षम नहीं होते हैं।
    • गलनांक अणुओं के बीच आकर्षण के अंतराआण्विक बल पर भी निर्भर करता है।
    • जब ठोस पिघलता है, तो इसके अणु मजबूत आणविक आकर्षण के विरुद्ध एक-दूसरे से दूर गति करते हैं। इसे ऊर्जा की आवश्यकता होती है जिसकी आपूर्ति बाहर से की जानी चाहिए।
    • अतः एक दिए गए निकाय की आंतरिक ऊर्जा ठोस चरण की तुलना में द्रव्य चरण में अधिक होती है।
    • उसीप्रकार, वाष्प चरण में दिए गए निकाय की आंतरिक ऊर्जा द्रव्य चरण की तुलना में अधिक होती है।
    Question:
    एक पुरानी और एक ताजा बनी अचार की दो बोतलों में से पुरानी बोतल को गंध और स्वाद से पहचाना जा सकता है जिसका कारण होता है:
    • 33% answered correctly
    • Your first attempt
    • Your first attempt
    Re-attempt answer
    Click here to see the answer now
    Reattempt mode is Off. Turn it on from bottom bar

    Your First Attempt Answers

  • Accepted answer is between: 0 and 0
  • सही उत्तर अचार में तेल का ऑक्सीकरण होने से अचार का खराब होना है।

    Key Points

    • विकृतगंधिता खाद्य पदार्थों में मौजूद तेल और वसा (जो वाष्पशील होते हैं) के धीमे ऑक्सीकरण की प्रक्रिया है जिसके परिणामस्वरूप उनमें गंध और स्वाद में परिवर्तन होता है।
    • विकृतगंधिता (बासीपन) को रोकने के लिए कई विधियां हैं:
      • आलू, पानी आदि खाद्य पदार्थों को वायु की जगह नाइट्रोजन गैस वाले पैकेट में पैक करना।
      • पके हुए भोजन और खाद्य सामग्री को सीधी धूप में रखने से बचें।
      • BHA (ब्यूटाइलेटेड हाइड्रॉक्सी एनीसोल) और BHT (ब्यूटाइलेटेड हाइड्रॉक्सी टोल्यूनि) जैसे प्रतिऑक्सीकारक को मिलाना।​

    Additional Information

    • सड़न जीवाणु और कवक द्वारा कार्बनिक पदार्थों के क्षय या अपघटन की एक प्रक्रिया है जिसमें दुर्गंधयुक्त अपूर्ण ऑक्सीकृत उत्पादों का निर्माण होता है।
    • किण्वन जीवाणु, यीस्ट या अन्य सूक्ष्मजीवों द्वारा किसी पदार्थ का रासायनिक विघटन है।
    Question:

    निम्नलिखित प्रश्न में दिए गए वाक्य में कुछ त्रुटियाँ हैं। त्रुटि वाले वाक्य के जिस भाग में त्रुटि हो, उसके अनुरूप अक्षर (A), (B), (C) को चुनिए। यदि वाक्य में कोई त्रुटि न हो, तो (D) का चुनाव कीजिए।

    एक भिखारिनी / घर के बाहर / खड़ी है / कोई त्रुटि नहीं

    • 68% answered correctly
    • Your first attempt
    • Your first attempt
    Re-attempt answer
    Click here to see the answer now
    Reattempt mode is Off. Turn it on from bottom bar

    Your First Attempt Answers

  • Accepted answer is between: 0 and 0
  • सही उत्तर 'एक भिखारिनीहोगा।

    Key Points  

    • दिए गए विकल्पों में 'एक भिखारिनी' यह विकल्प अनुचित है।
    • 'एक भिखारिनी' के स्थान पर 'एक भिखारिन' सही होगा। 
    • शुद्ध वाक्य होगा - एक भिखारिन दरवाजे पर खड़ी थी। 

    Additional Information   

    • शुद्ध भाषा के प्रयोग के लिए वर्णों के शुद्ध उच्चारण, शब्दों के शुद्ध रूप और वाक्यों के शुद्ध रूप जानना आवश्यक है।
    • वही शुद्ध वाक्य वाक्य होते हैं जिसमें व्याकरण संबंधी त्रुटियां नहीं होती हैं तथा या पढ़ने में भी आसान होते हैं एवं इन्हें आसानी से समझा जा सकता है। 
    Question:
    सूचना का अधिकार (RTI) पहली बार किस वर्ष में पेश किया गया था?
    • 61% answered correctly
    • Your first attempt
    • Your first attempt
    Re-attempt answer
    Click here to see the answer now
    Reattempt mode is Off. Turn it on from bottom bar

    Your First Attempt Answers

  • Accepted answer is between: 0 and 0
  • सही उत्तर 2005 है।

    Important Points

    • सूचना का अधिकार एक उपकरण है, जिसके साथ भारत में प्रत्येक व्यक्ति ऐसी जानकारी प्राप्त कर सकता है जो उनके जीवन को बेहतर बना सकती है।
    • भारत ने वर्ष 2005 में सूचना का अधिकार अधिनियम लागू करने की पहल की।
    • सूचना का अधिकार अधिनियम 2005 सरकारी सूचना के लिए नागरिकों के अनुरोधों पर समय पर प्रतिक्रिया देता है।
    • यह कार्मिक और प्रशिक्षण विभाग, कार्मिक, लोक शिकायत और पेंशन मंत्रालय द्वारा की गई एक पहल है।
    • सूचना का अधिकार अधिनियम का मुख्य उद्देश्य नागरिकों को सशक्त बनाना, सरकार के काम में पारदर्शिता और जवाबदेही को बढ़ावा देना, भ्रष्टाचार को नियंत्रित करना और सही मायने में लोगों की जरूरतों को पूरा करवाना है।
    • सूचना का अधिकार अधिनियम पारित करने वाला स्वीडन पहला देश है।

    Key Points

    • ​सूचना का अधिकार (RTI) पहली बार वर्ष 2005 में पेश किया गया था।
    • इस अधिनियम को 15 जून 2005 को राष्ट्रपति की स्वीकृति मिली।
    • भारत के पूर्व राष्ट्रपति एपीजे अब्दुल कलाम ने राष्ट्रीय सूचना अधिकार अधिनियम 2005 को अपनी स्वीकृति दी।
    • 12 अक्टूबर 2005 को RTI अधिनियम लागू हुआ।
    • आरटीआई अधिनियम 2005 में केवल 2 अनुसूचियाँ हैं।
    • मजदूर किसान शक्ति संगठन आरटीआई अधिनियम के पारित होने में सहायक संगठन है।
    • भारत में पहला आरटीआई आवेदन शाहिद रज़ा बर्नी द्वारा प्रस्तुत किया गया था।
    Question:
    कोशिका निम्नलिखित भागों में से कौन से आवरण से बंधा हुआ नहीं है?
    • 21% answered correctly
    • Your first attempt
    • Your first attempt
    Re-attempt answer
    Click here to see the answer now
    Reattempt mode is Off. Turn it on from bottom bar

    Your First Attempt Answers

  • Accepted answer is between: 0 and 0
  • सही उत्तर विकल्प 2, अर्थात रायबोझोम है।

    • रायबोझोम राइबोसोमल आरएनए और प्रथिन के परिसर हैं।
      • रायबोझोम प्रथिन संश्लेषण करते हैं।
      • रायबोझोम को अंगक नहीं माना जाता है क्योंकि उनके आवरण नहीं होते हैं।
    • केंद्रक एक केंद्रक आवरक से घिरा होता है।
      • केंद्रक आवरक एक दोहरी आवरण है।
    • रिक्तिका बड़े पीठिका होती हैं जो गॉल्गी तंत्र और आंतरर्द्रव्यजालिका से प्राप्त होती हैं।
      • रिक्तिका में आवरण होती है।
      • रिक्तिका आवरण विलेय के परिवहन में चयनात्मक है।
    • लयकायिका हाइड्रोलाइटिक प्रकिण्वों का एक आवरण थैली है।
      • लयकायिका आवरण रूक्ष आंतरद्रव्यजालिका द्वारा बनाई जाती है और फिर आगे की प्रक्रिया के लिए गोल्गी तंत्र में स्थानांतरित हो जाती है।
    Question:
    पिता की वर्तमान आयु का उसके पुत्र की आयु से अनुपात 7 : 2 है। यदि 10 वर्ष बाद उनकी आयु का अनुपात 9 : 4 हो जाएगा, तो पिता की वर्तमान आयु क्या है?
    • 74% answered correctly
    • Your first attempt
    • Your first attempt
    Re-attempt answer
    Click here to see the answer now
    Reattempt mode is Off. Turn it on from bottom bar

    Your First Attempt Answers

  • Accepted answer is between: 0 and 0
  • दिया गया है:

    पिता की वर्तमान आयु का उसके पुत्र की आयु से अनुपात = F : S =  7 : 2

    गणना:

    माना, पिता की वर्तमान आयु (F) = 7x

    पुत्र की वर्तमान आयु (S) = 2x

    प्रश्न के अनुसार:

    ⇒ (7x + 10)/(2x + 10) = 9/4

    ⇒ 28x + 40 = 18x + 90

    ⇒ 28x - 18x = 90 - 40

    ⇒ 10x = 50 

    ⇒ x = 5

    इसलिए, पिता की वर्तमान आयु (F) = 7x = 7 × 5 = 35 वर्ष

    ∴ सही उत्तर 35 वर्ष है।

    Question:
    भारत में राष्ट्रीय महिला आयोग की स्थापना किस वर्ष हुई थी?
    • 55% answered correctly
    • Your first attempt
    • Your first attempt
    Re-attempt answer
    Click here to see the answer now
    Reattempt mode is Off. Turn it on from bottom bar

    Your First Attempt Answers

  • Accepted answer is between: 0 and 0
  • सही उत्तर 1992 है।

    Key Points

    • राष्ट्रीय महिला आयोग की स्थापना न्यायिक और संवैधानिक परिवर्तनों के माध्यम से भारतीय महिलाओं के लिए एक समान और न्यायपूर्ण जीवन स्तर स्थापित करने के लक्ष्य के साथ की गई थी।
    • 31 जनवरी 1992 को, पहला आयोग स्थापित किया गया था, जिसमें जयंती पटनायक अध्यक्ष के रूप में कार्यरत थे। राष्ट्रीय महिला आयोग के पहले पुरुष सदस्य आईएएस आलोक रावत (NCW) हैं। उनके चयन ने निकाय के पांच सदस्यों में चौथे स्थान पर कब्जा कर लिया।
    • इस समय राष्ट्रीय महिला आयोग की अध्यक्षा सुश्री रेखा शर्मा हैं। सितंबर 2018 में, उन्होंने ललिता कुमारमंगलम को नए अध्यक्ष के रूप में स्थान दिया।​

    Additional Information

    • आयोग में अध्यक्ष, सदस्य सचिव और शेष पांच सदस्यों सहित कम से कम छह सदस्य होने चाहिए।
      अध्यक्ष को राष्ट्रीय सरकार द्वारा चुना जाना चाहिए।
    • सदस्य सचिवों को भी केंद्र सरकार द्वारा चुना जाता है। वह संगठन का सदस्य, सदस्य संगठन या प्रबंधन विशेषज्ञ होना चाहिए।
    • एनसीडब्ल्यू के सदस्य महिलाओं के सामाजिक आर्थिक विकास की योजना में भाग लेते हैं, सभी क्षेत्रों में महिलाओं के प्रतिनिधित्व को बढ़ावा देने के लिए नीतियों का सुझाव देते हैं और महिलाओं के विकास का आकलन करते हैं।​​
    Question:
    अलवर संतों की उत्पत्ति निम्नलिखित में से किस राज्य में हुई थी?
    • 16% answered correctly
    • Your first attempt
    • Your first attempt
    Re-attempt answer
    Click here to see the answer now
    Reattempt mode is Off. Turn it on from bottom bar

    Your First Attempt Answers

  • Accepted answer is between: 0 and 0
  • सही उत्तर तमि‍लनाडु हैं।

    Key Points

    • अलवर संत तमिलनाडु के प्रसिद्ध तमिल कवि संत थे।
    • अलवर संतों की उत्पत्ति दक्षिण भारत के तमिल भाषी क्षेत्र, तमिलनाडु से हुई है।
    • माना जाता है कि वे 4200 ईसा पूर्व - 2700 ईसा पूर्व के बीच रहे थे।
    • अलवर शब्द का अर्थ है "वह जो भगवान के अनुभव में निमग्न है",
    • उन्होंने लगभग 4000 तमिल छंदों की रचना की।
    • अलवर संतों ने अपने स्वयं के रचित गीतों के साथ भगवान विष्णु और उनके अवतार कृष्ण की प्रशंसा की।
    • वे भगवान विष्णु की आराधना में परमानंद के भजन गाते हुए मंदिर-मंदिर घूमते थे।
    • 12 अलवर हैं, उनमें से केवल श्री अंडाल ही एक महिला है।
    • अलवर के भजनों को एक संगठित आवाज़ में बनाया गया था जिसे दिव्य प्रबंध के नाम से जाना जाता है।
    Question:
    निम्न में से कौन-से जंतु कूटगुहिक (pseudocoelomate) नहीं होते हैं?
    • 32% answered correctly
    • Your first attempt
    • Your first attempt
    Re-attempt answer
    Click here to see the answer now
    Reattempt mode is Off. Turn it on from bottom bar

    Your First Attempt Answers

  • Accepted answer is between: 0 and 0
  • सही उत्तर बिच्छू है।

    Key Points

    • प्रगुहा (coelom) देह भित्ति और आंत्र भित्ति के बीच द्रव से भरी एक गुहा होती है।
    • प्रगुहा मध्यजनस्तर (मीजोडर्म) द्वारा आस्तरित होती है।
    • प्रगुहा उत्तरजंतु (मेटाज़ोअन) की विशिष्ट विशेषताओं में से एक है।
    • प्रगुहा की उपस्थिति या अनुपस्थिति के आधार पर जीवों को तीन समूहों में वर्गीकृत किया गया है:
      • प्रगुहीय (Coelomates)
      • कूटगुहिक (Pseudocoelomates)
      • अगुहिय (Acoelomates)
    • कूटगुहिकों में, देह भित्ति मध्यजनस्तर द्वारा आस्तरित नहीं होती है।
    • एस्केल्मिन्थीज कूटप्रगुहिक जीव होते हैं।

    Additional Information

    • प्रगुहा अधिकांश जंतुओं में प्रमुख देह गुहा है, जो आंत्र नालिका और देह भित्ति के बीच स्थित होती है।
    • प्रगुही जीव मध्यजनस्तर के भीतर स्थित तरल पदार्थ से भरे प्रगुही नामक देह गुहा वाले जंतुओं को संदर्भित करता है। लघुवलयक कीट, मृदुकवची जीव, संधिपाद और शल्यचर्मी जीव प्रगुही जीवों के कुछ उदाहरण हैं।
    • कूटगुहा का अर्थ "बनावटी प्रगुहा" या "बनावटी गुहा" है।
    Question:
    पाटलिपुत्र शहर की स्थापना किसके द्वारा की गई थी?
    • 78% answered correctly
    • Your first attempt
    • Your first attempt
    Re-attempt answer
    Click here to see the answer now
    Reattempt mode is Off. Turn it on from bottom bar

    Your First Attempt Answers

  • Accepted answer is between: 0 and 0
  • सही उत्‍तर है → अजातशत्रु।

    प्रमुख बिंदु

    • पाटलिपुत्र का प्राचीन शहर 5 वीं शताब्दी ईसा पूर्व में मगध (दक्षिण बिहार) के राजा अजातशत्रु द्वारा स्थापित किया गया था।
    • उनके पुत्र उदय (उदयिन) ने इसे मगध की राजधानी बनाया, जो पहली शताब्दी ईसा पूर्व तक बना रहा।
    • उदयिन ने अपनी राजधानी राजगीर से पाटलिपुत्र स्थानांतरित कर दी।
    • अजातशत्रु का पुत्र उदयिन, हर्यंका वंश का एक शासक था जिसने 460 ईसा पूर्व से 444 ईसा पूर्व तक मगध पर शासन किया था।
    • बाद में इसे मगध साम्राज्य की राजधानी बनाया गया।
    • पहले यह राजगृह था।
    • उदयिन हर्यंका राज्य के अजातशत्रु का पुत्र था।
    • यह तीसरी से छठी शताब्दी तक गुप्त वंश की राजधानी भी थी, 8 वीं से 12 वीं शताब्दी तक पाल वंश और 16 वीं शताब्दी में शेरशाह सूरी का।
    • ग्रीक भूगोलवेत्ता स्ट्रैबो ने अपनी पुस्तक ' जियोग्राफिका ' में शहर का उल्लेख किया है।
    • उदयिन ने 460-444 ई.पू. तक शासन किया।
    Question:
    निम्नलिखित में से किस खिलाड़ी ने पुरुष IPL 2023 में ऑरेंज कैप जीती?
    • 76% answered correctly
    • Your first attempt
    • Your first attempt
    Re-attempt answer
    Click here to see the answer now
    Reattempt mode is Off. Turn it on from bottom bar

    Your First Attempt Answers

  • Accepted answer is between: 0 and 0
  • सही उत्तर शुभमन गिल है।

    Key Points 

    • इंडियन प्रीमियर लीग (IPL) 2023 का समापन चेन्नई सुपर किंग्स द्वारा अहमदाबाद के नरेंद्र मोदी स्टेडियम में गत चैंपियन गुजरात टाइटंस को हराकर ट्रॉफी उठाने के साथ हुआ।
    • इंडियन प्रीमियर लीग (IPL) 2023, भारतीय क्रिकेट कंट्रोल बोर्ड (BCCI) द्वारा स्थापित पेशेवर ट्वेंटी 20 क्रिकेट लीग का 16वाँ संस्करण था।
    • ऑरेंज कैप खिताब विजेता शुभमन गिल (गुजरात टाइटन्स) हैं, जिनके कुल रन 890 हैं।
    • पर्पल कैप खिताब विजेता मोहम्मद शमी (गुजरात टाइटन्स) हैं, जिन्होंने कुल 28 विकेट लिए हैं।

    Additional Information 

    • अन्य पुरस्कार और मान्यताएँ:
      • ईडन गार्डन्स और वानखेड़े स्टेडियम ने सीज़न के सर्वश्रेष्ठ स्थानों का पुरस्कार साझा किया।
      • दिल्ली कैपिटल्स ने फ़ेयरप्ले ऑफ़ द सीज़न का पुरस्कार जीता।
      • राशिद खान ने कैच ऑफ द सीजन का पुरस्कार जीता।
      • फाफ डु प्लेसिस ने सीजन का सबसे लंबा छक्का (115 मीटर) का पुरस्कार जीता।
      • शुभमन गिल ने सर्वाधिक चौके (84) का पुरस्कार जीता।
      • शुभमन गिल ने सीजन के सबसे मूल्यवान खिलाड़ी का पुरस्कार जीता।
      • शुभमन गिल ने ड्रीम11 गेमचेंजर ऑफ द सीजन का पुरस्कार जीता।
      • ग्लेन मैक्सवेल ने सुपर स्ट्राइकर ऑफ़ द सीज़न का पुरस्कार जीता।
      • यशस्वी जयसवाल ने इमर्जिंग प्लेयर ऑफ द ईयर का पुरस्कार जीता।
    Question:
    एक मोबाइल फोन चार्जर _______ है
    • 43% answered correctly
    • Your first attempt
    • Your first attempt
    Re-attempt answer
    Click here to see the answer now
    Reattempt mode is Off. Turn it on from bottom bar

    Your First Attempt Answers

  • Accepted answer is between: 0 and 0
  • सही उत्तर एक अपचायी ट्रांसफार्मर है।

    Key Points

    • मोबाइल फोन चार्जर एक अपचायी ट्रांसफार्मर है।
      • मोबाइल फोन चार्जर में 220 V के इनपुट को लगभग 5 V के कम करने वाले आउटपुट वोल्टेज में बदलने के लिए एक अपचायी ट्रांसफार्मर होता है।
      • एक ट्रांसफार्मर में कॉपर वाइंडिंग।
      • यदि द्वितीयक कुंडल में प्राथमिक कुंडल की तुलना में अधिक मोड़ हैं तो आउटपुट वोल्टेज इनपुट वोल्टेज से बड़ा है।
      • एक मोबाइल बैटरी चार्जर एक प्रकार की बिजली आपूर्ति है जिसे स्विच्ड मोड शक्ति आपूर्ति कहा जाता है।
      • यह बैटरी को एक स्थिर वोल्टेज (CV) और एक स्थिर विधुत धारा (CC) प्रदान करता है, इसे प्रदान करने के लिए आवश्यक ऊर्जा प्रदान करता है।
      • आम तौर पर, वोल्टेज 5V DC है और विधुत धारा की 0.5 A या 1 A है।
    • इस प्रकार, विकल्प 4 सही है।

    Additional Information

    • एक मोबाइल चार्जर AC (प्रत्यावर्ती धारा) से DC (दिष्ट धारा) के रूपांतरण के सरल सिद्धांत पर काम करता है।
    • 5V आउटपुट को पूरा करने के लिए स्मार्टफोन के लिए ठीक वैसा ही आवश्यक था जैसा कि उसे चरणबद्ध तरीके से काम करना है।
    • स्टेप डाउनिंग:
      • 5V आउटपुट का उत्पादन करने के लिए, इसे एक इनपुट वोल्टेज की आवश्यकता होती है।
      • वह इनपुट वोल्टेज 220 V AC है जो एक घर में इस्तेमाल होने वाला सामान्य घरेलू मान है।
      • 220 V को 9V या अन्य ऑपरेटिंग कम वोल्टेज में किया गया है, जिस पर दिष्टकारी अपने सबसे अच्छे तरीके से काम कर सकता है।
    • रेक्टिफाईंग:
      • यह वह प्रक्रिया है जो AC वोल्टेज को DC में परिवर्तित करती है।
      • लेकिन रूपांतरण 100% नहीं किया जाएगा।
      • रूपांतरण में अपूर्णता होगी जो हमेशा एक व्यावहारिक सीमा के रूप में होती थी जिसे आगामी चरणों में मंजूरी दे दी जाएगी।
    • स्मूथिंग या फ़िल्टरिंग:
      • परिवर्तित डीसी वोल्टेज में कुछ तरंग होते हैं जिसका अर्थ है कि यह कुछ AC विशेषता रखता हैं जो संयोजन प्रेरक और संधारित्र फ़िल्टर परिपथ द्वारा फ़िल्टर किया जाएगा।
      • इस प्रक्रिया का आउटपुट लगभग उम्मीद के मुताबिक होगा।
    • नियामक:
      • रिपल्स के बिना DC को प्राप्त करने के बाद, 9V मान को 5V तक काटना होगा।
      • ऐसा करने के लिए, नियामक एक वोल्टेज जगह पर आता है
      • यह 9V इनपुट के लिए आउटपुट के रूप में लगातार 5V आपूर्ति देता है।
    • लोड (स्मार्टफोन):
      • यह कुल प्रक्रिया का लाभार्थी है।
      • बैटरी ऊर्जा को भरने के लिए स्मार्टफोन के अंदर इलेक्ट्रॉनों को ले जाने के लिए उस चार्जर द्वारा 5V आउटपुट का उपयोग किया जाता है।
    Question:
    यदि A का 80% = B का 50% और B = A का x%, तो x का मान है
    • 63% answered correctly
    • Your first attempt
    • Your first attempt
    Re-attempt answer
    Click here to see the answer now
    Reattempt mode is Off. Turn it on from bottom bar

    Your First Attempt Answers

  • Accepted answer is between: 0 and 0
  • दिया गया है:

    A का 80% = B का 50% और B = A का x%

    गणना:

    ⇒ A × 80 / 100 = B × 50 / 100

    ⇒ B = (A × 80) / 50 = 1.6A

    ⇒ B = A का 160%

    x का आवश्यक मान = 160

    Question:
    निम्नलिखित में से कौन सा न्यायालय किसी अदालत का मुकदमा अपने पास मंगवा सकता है?
    • 82% answered correctly
    • Your first attempt
    • Your first attempt
    Re-attempt answer
    Click here to see the answer now
    Reattempt mode is Off. Turn it on from bottom bar

    Your First Attempt Answers

  • Accepted answer is between: 0 and 0
    • भारत के सर्वोच्च न्यायालय के पास किसी भी अदालत से किसी मामले को स्वयं स्थानांतरित करने का अधिकार है।
    • यह भारतीय संविधान के अनुच्छेद 139 ए के अनुसार एससी द्वारा प्रयोग किया जाता है।
    • यह तब किया जा सकता है जब संबंधित मामले में कानून का कोई ठोस प्रश्न शामिल हो।
    Question:
    किस महाद्वीप को 'अंध' महाद्वीप के रूप में जाना जाता है?
    • 71% answered correctly
    • Your first attempt
    • Your first attempt
    Re-attempt answer
    Click here to see the answer now
    Reattempt mode is Off. Turn it on from bottom bar

    Your First Attempt Answers

  • Accepted answer is between: 0 and 0
  • सही उत्तर है अफ्रीका

    Key Points

    • अफ्रीका को 'अंध' महाद्वीप के रूप में जाना जाता है।
    • क्योंकि लंबे समय तक इसकी ख़ोज नहीं की जा सकी थी।
    • अफ्रीका में सबसे बड़ा मरुस्थल, उच्च पठारी क्षेत्र शामिल हैं, यहाँ बहुत अधिक प्राकृतिक बंदरगाह नहीं हैं तथा जलवायु ऊष्ण और आर्द्र है, जिसने विदेशियों को महाद्वीप में प्रवेश करने और पता लगाने में असमर्थ बना दिया।
    • यूरोपवासी इस महाद्वीप के आंतरिक क्षेत्र का पता लगाने में सक्षम नहीं थे और इसकी रहस्यमय प्रकृति के कारण उन्होंने इसे 'अंध' महाद्वीप नाम दिया।
    Question:
    शुंग वंश के संस्थापक कौन थे?
    • 83% answered correctly
    • Your first attempt
    • Your first attempt
    Re-attempt answer
    Click here to see the answer now
    Reattempt mode is Off. Turn it on from bottom bar

    Your First Attempt Answers

  • Accepted answer is between: 0 and 0
  • सही उत्‍तर है → पुष्यमित्र

    • शुंग राजवंश प्राचीन भारत का एक राजवंश था जिसने मौर्य वंश के बाद शासन किया था।

    प्रमुख बिंदु

    • उन्होंने 184 ईस्वी से 75 ईसा पूर्व तक शासन किया।
    • इनकी राजधानी पाटलिपुत्र थी
    • इसकी स्थापना पुष्यमित्र ने की थी। दस शुंग शासन थे।
    • पुष्यमित्र पिछले मौर्य सम्राट Vrihadratha का कमांडर था।
    • पुष्यमित्र ने अंतिम सम्राट वृहद्रता को मार डाला और पूरे मगध पर कब्जा कर लिया।
    • मगध के मौर्य वंश के पहले शासक का नाम भी वृहद्रथ था और अंतिम शासक का नाम भी वृहद्रथ था।

    अतिरिक्त जानकारी

    • शुंग वंश ने 185-75 ईसा पूर्व तक पूर्वी भारत, बांग्लादेश और नेपाल पर शासन किया।
    • मौर्य साम्राज्य के पतन के बाद पुष्यमित्र शुंग ने इसकी स्थापना की थी।
    • इसकी राजधानी पाटलिपुत्र थी , लेकिन बाद के शासकों ने बेसनगर (मध्य प्रदेश में आधुनिक विदिशा) में भी दरबार लगाया।

    महत्वपूर्ण बिंदु

    • शुंगों के पूर्वजों की उत्पत्ति मध्य प्रदेश के उज्जैन शहर से हुई थी। साहित्यिक और पुरातत्व दोनों साक्ष्य हैं जो दिखाते हैं कि शुंग राजवंश के संस्थापक और पहले राजा उज्जैन में मौर्य के वाइसराय थे और उनके पूर्वज उज्जैन के थे।
    • मगध दक्षिणी बिहार में एक प्राचीन भारतीय साम्राज्य था।
    • मगध शुंग साम्राज्य का केंद्र था।
    • शुंग राजवंश (185 ईसा पूर्व से 73 ईसा पूर्व)
      • Shunga राजवंश पुष्यमित्र जो अंतिम मौर्य सम्राट Brihadratha हत्या कर दी और मौर्य वंश की जगह द्वारा स्थापित किया गया।
      • वह उज्जैन में मौर्य का वायसराय था।
      • शुंग वंश की राजधानी पाटलिपुत्र थी   और इसके प्रमुख केंद्र उज्जैन, मथुरा, साकेत, सांची और कपिलवस्तु थे।
      • विदिशा बाद के शुंग शासकों की राजधानी थी।
      • शुंग वंश के अन्य शासक:-
        • अग्निमित्र (149 ईसा पूर्व से 141 ईसा पूर्व)।
        • भागभद्रा (लगभग 110 ई.पू.)
        • देवभूति (87 ईसा पूर्व से 73 ईसा पूर्व)।
    Question:
    धार्मिक संस्कारों से सम्बंधित वेद है________
    • 26% answered correctly
    • Your first attempt
    • Your first attempt
    Re-attempt answer
    Click here to see the answer now
    Reattempt mode is Off. Turn it on from bottom bar

    Your First Attempt Answers

  • Accepted answer is between: 0 and 0
  • सही उत्तर यजुर्वेद है।

    वेद

    • वेद भारतीय उपमहाद्वीप का सबसे पुराना जीवित साहित्य है।
    • चार वेद हैं: ऋग्वेद, सामवेद, यजुर्वेद और अथर्ववेद।

    Important Points यजुर्वेद:

    • 'पूजा ज्ञान' का अर्थ है, यजुर्वेद 1100-800 ईसा में वर्णित है तथा सामवेद में भी इसके समान  है।
    • यह अनुष्ठान-अर्पण मंत्रों / मंत्रों का संकलन करता है। ये मंत्र पुजारी द्वारा एक ऐसे व्यक्ति के साथ पेश किए जाते थे जो एक अनुष्ठान करता था (ज्यादातर मामलों में यज्ञ अग्नि।)
    • इसके दो प्रकार हैं - कृष्ण (काला/गहरा) और शुक्ल (सफेद/उज्ज्वल)
    • कृष्ण यजुर्वेद में छंदों का एक अव्यवस्थित, अस्पष्ट, प्रेरक संग्रह है। 
    • शुक्ल यजुर्वेद ने श्लोकों को व्यवस्थित और स्पष्ट किया है।

    इस प्रकार, हम कह सकते हैं कि धार्मिक संस्कारों से सम्बंधित वेद यजुर्वेद है।

    Additional Information

    • ऋग्वेद:
      • सबसे प्राचीन वेद ऋग्वेद है। इसमें 1028 सूक्त हैं जिन्हें 'सूक्त' कहा जाता है और यह 10 पुस्तकों का संग्रह है जिसे  'मंडल' कहते हैं।
      • यह सबसे पुराना वेद है और सबसे पुराना ज्ञात वैदिक संस्कृत पाठ (1800 - 1100 ईसा पूर्व) है।
      • 'ऋग्वेद' शब्द का अर्थ स्तुति ज्ञान है। 
      • इसमें 10600 श्लोक हैं। 
    • सामवेद:
      • धुनों और मंत्रों के वेद के रूप में जाना जाने वाला सामवेद 1200-800 ईसा पूर्व का है। इस वेद का संबंध लोक पूजा से है।
      • इसमें 1549 श्लोक हैं (75 श्लोकों को छोड़कर सभी ऋग्वेद से लिए गए हैं)
      • सामवेद में दो उपनिषद सन्निहित हैं - छांदोग्य उपनिषद और केनोपनिषद 
      • सामवेद को भारतीय शास्त्रीय संगीत और नृत्य का मूल माना जाता है। 
      • इसे मधुर मंत्रों का भण्डार माना जाता है। 
    • अथर्ववेद:
      • अथर्वन, एक प्राचीन ऋषि, और ज्ञान (अथर्वन + ज्ञान) का एक तत्पुरुष यौगिक है, यह 1000-800 ईसा पूर्व का है।
      • इस वेद में जीवन की दैनिक प्रक्रियाओं का बहुत अच्छी तरह से वर्णन किया गया है
      • इसमें 730 सूक्त, 6000 मंत्र और 20 पुस्तकें हैं। 
      • पैप्पलाद और सौनाकिया अथर्ववेद के दो जीवित अंश हैं। 
      • जादुई सूत्रों का वेद कहा जाता है, इसमें तीन प्राथमिक उपनिषद शामिल हैं - मुंडक उपनिषद, मांडुक्य उपनिषद, और प्रश्न उपनिषद I
    Question:
    The word opposite in meaning to 'Kind' is
    • 60% answered correctly
    • Your first attempt
    • Your first attempt
    Re-attempt answer
    Click here to see the answer now
    Reattempt mode is Off. Turn it on from bottom bar

    Your First Attempt Answers

  • Accepted answer is between: 0 and 0
  • The correct answer is "Cruel".

    Key Points

    •  'Kind' means generous, helpful, and thinking about other people's feelings.(उदार, मददगार और दूसरे लोगों की भावनाओं के बारे में सोचने वाला)
      • Example: One should be kind towards the poor of society.
    • 'Cruel' means someone extremely unkind and causing pain to people or animals.(लोगों या जानवरों को पीड़ा पहुँचाने वाला)
      • Example: "The king's cruel reign caused great suffering."
    • It is clear from the above points, that 'Cruel' is the antonym of 'Kind'.
    • Hence, 'Option 4' is the correct answer.

    Additional Information

    • Let's have a look at the meaning of the other words-
    • Famous- someone known and recognized by many people.(ऐसा व्यक्ति जिसे बहुत से लोग जानते हों और पहचानते हों)
    • Similar- two things or persons having the same qualities, characteristics, or appearance.(दो चीजें या व्यक्ति जिनके गुण, विशेषताएँ या रूप एक जैसे हों)
    • Honest-  good and truthful, and not lying to anyone.(अच्छा और सच्चा, और किसी से झूठ नहीं बोलना)

    Post a Comment

    0 Comments
    * Please Don't Spam Here. All the Comments are Reviewed by Admin.